48
MyPasTest » MRCS A Online - Jan Exam 2015 14. Principles of Surgery in General; Perioperative Care (55Qs) ------------------------------------------------------------------------------------------------------------------- --------------------------------------------------------------------------------------------------------------------------------------------------------------------------------------------------------------- Dr Mohammed Shamsul Islam Khan, Medical Officer, Clinical Neuro-Surgery, National Institute of Neuro-Sciences and Hospital Sher-E-Bangla Nagar, Dhaka-1207, Bangladesh. Mobile: +880 1713 455 662, +880 1685 811979. E-mail: [email protected] Page | 1104 01. THEME: Local analgesia and procedures A 1% Lignocaine B 1% Lignocaine with adrenaline C Preservative-free prilocaine D 0.5% Bupivacaine with adrenaline E Amethocaine F Xylocaine® spray. For each of the cases described below, select the single most appropriate anaesthetic from the options listed above. Each option may be used once, more than once, or not at all. Scenario 1 Inguinal hernia repair. A - 1% Lignocaine« YOUR ANSWER B - 1% Lignocaine with adrenaline« CORRECT ANSWER. The local anaesthetic of choice for an inguinal hernia repair would be lignocaine with adrenaline as this combines a rapid onset of response together with reducing bleeding. Scenario 2 Bier’s block. B - 1% Lignocaine with adrenaline« YOUR ANSWER C - Preservative-free prilocaine« CORRECT ANSWER. Prilocaine is similar to lignocaine and it is useful when a large amount of local anaesthetic is needed or a high plasma concentration is likely. Its particular use is in intravenous regional anaesthesia, ie a Bier’s block. Scenario 3 Digital nerve (ring) block. C - Preservative-free prilocaine« YOUR ANSWER A - 1% Lignocaine« CORRECT ANSWER. Digital nerve blocks are usually performed using lignocaine. It is important not to use any vasoconstrictor agents such as adrenaline as this may cause distal gangrene. Scenario 4 Excision scalp lesion. D - 0.5% Bupivacaine with adrenaline« YOUR ANSWER B - 1% Lignocaine with adrenaline« CORRECT ANSWER. This would give good analgesia. And the adrenaline will help minimize bleeding. Scenario 5 Manipulation of Colles’ fracture. E - Amethocaine« YOUR ANSWER C - Preservative-free prilocaine« CORRECT ANSWER.

Page | 1104

  • Upload
    others

  • View
    10

  • Download
    0

Embed Size (px)

Citation preview

Page 1: Page | 1104

MyPasTest » MRCS A Online - Jan Exam 2015 14. Principles of Surgery in General; Perioperative Care (55Qs)

-------------------------------------------------------------------------------------------------------------------

---------------------------------------------------------------------------------------------------------------------------------------------------------------------------------------------------------------

Dr Mohammed Shamsul Islam Khan, Medical Officer, Clinical Neuro-Surgery, National Institute of Neuro-Sciences and Hospital

Sher-E-Bangla Nagar, Dhaka-1207, Bangladesh. Mobile: +880 1713 455 662, +880 1685 811979. E-mail: [email protected]

Page |

1104

01. THEME: Local analgesia and procedures A 1% Lignocaine B 1% Lignocaine with adrenaline C Preservative-free prilocaine D 0.5% Bupivacaine with adrenaline E Amethocaine F Xylocaine® spray. For each of the cases described below, select the single most appropriate anaesthetic from the options listed above. Each option may be used once, more than once, or not at all. Scenario 1 Inguinal hernia repair. A - 1% Lignocaine« YOUR ANSWER B - 1% Lignocaine with adrenaline« CORRECT ANSWER. The local anaesthetic of choice for an inguinal hernia repair would be lignocaine with adrenaline as this combines a rapid onset of response together with reducing bleeding. Scenario 2 Bier’s block. B - 1% Lignocaine with adrenaline« YOUR ANSWER C - Preservative-free prilocaine« CORRECT ANSWER. Prilocaine is similar to lignocaine and it is useful when a large amount of local anaesthetic is needed or a high plasma concentration is likely. Its particular use is in intravenous regional anaesthesia, ie a Bier’s block. Scenario 3 Digital nerve (ring) block. C - Preservative-free prilocaine« YOUR ANSWER A - 1% Lignocaine« CORRECT ANSWER. Digital nerve blocks are usually performed using lignocaine. It is important not to use any vasoconstrictor agents such as adrenaline as this may cause distal gangrene. Scenario 4 Excision scalp lesion. D - 0.5% Bupivacaine with adrenaline« YOUR ANSWER B - 1% Lignocaine with adrenaline« CORRECT ANSWER. This would give good analgesia. And the adrenaline will help minimize bleeding. Scenario 5 Manipulation of Colles’ fracture. E - Amethocaine« YOUR ANSWER C - Preservative-free prilocaine« CORRECT ANSWER.

Page 2: Page | 1104

MyPasTest » MRCS A Online - Jan Exam 2015 14. Principles of Surgery in General; Perioperative Care (55Qs)

-------------------------------------------------------------------------------------------------------------------

---------------------------------------------------------------------------------------------------------------------------------------------------------------------------------------------------------------

Dr Mohammed Shamsul Islam Khan, Medical Officer, Clinical Neuro-Surgery, National Institute of Neuro-Sciences and Hospital

Sher-E-Bangla Nagar, Dhaka-1207, Bangladesh. Mobile: +880 1713 455 662, +880 1685 811979. E-mail: [email protected]

Page |

1105

Scenario 6 Post-operative analgesia following inguinal hernia repair. F - Xylocaine® spray« YOUR ANSWER D - 0.5% Bupivacaine with adrenaline« CORRECT ANSWER. Bupivacaine is a long acting amide local anaesthetic which can be used for post-operative analgesia following inguinal hernia repair. 02. A 34-year-old patient undergoes bilateral breast reduction. During the operation, her arms are placed out in supine position. The theatre assistant forgets to place a gel pad. Which nerve is at risk of compression at the level of the elbow?

Axillary« YOUR ANSWER

Median

Musculocutaneous

Radial

Ulnar« CORRECT ANSWER. The ulnar nerve is at risk of compression at the elbow. 03. Theme: Coagulation disorders A Antithrombin III deficiency B Disseminated intravascular coagulation (DIC) C Factor V Leiden deficiency D Haemophilia A E Haemophilia B F Idiopathic thrombocytopenia purpura (ITP) G Protein C deficiency H Protein S deficiency I Thrombasthenia (Glanzmann’s disease) J Thrombocytosis K Vitamin K deficiency L Von Willebrand’s disease. The terms above all refer to types of coagulation disorders. Please select the most appropriate term from this list for the scenarios below. The items may be used once, more than once, or not at all. Scenario 1 A 45-year-old woman, with a vague history of pulmonary embolism (PE), has developed a deep vein thrombosis (DVT) following a laparoscopic cholecystectomy. She is currently taking warfarin but has begun to notice multiple areas of skin necrosis on her body. A - Antithrombin III deficiency« YOUR ANSWER G - Protein C deficiency« CORRECT ANSWER. Protein C stimulates fibrinolysis by destroying activated factors in the coagulation pathway. Its deficiency leads to unchecked coagulation and is the commonest form of hereditary thrombophilia. It is inherited in

Page 3: Page | 1104

MyPasTest » MRCS A Online - Jan Exam 2015 14. Principles of Surgery in General; Perioperative Care (55Qs)

-------------------------------------------------------------------------------------------------------------------

---------------------------------------------------------------------------------------------------------------------------------------------------------------------------------------------------------------

Dr Mohammed Shamsul Islam Khan, Medical Officer, Clinical Neuro-Surgery, National Institute of Neuro-Sciences and Hospital

Sher-E-Bangla Nagar, Dhaka-1207, Bangladesh. Mobile: +880 1713 455 662, +880 1685 811979. E-mail: [email protected]

Page |

1106

an autosomal dominant pattern with a variable penetrance. Patients can develop skin necrosis when treated with warfarin because it triggers protein C levels to drop further before the therapeutic inhibition of vitamin K-dependent clotting factors occurs. Scenario 2 A 34-year-old man undergoes a total colectomy with ileostomy for severe refractory ulcerative colitis. He develops a post-operative septicaemia and is transferred to the Intensive Care Unit where intravenous antibiotic therapy is commenced. Nine days following transfer, he is noted to develop purpura and begins oozing from venepuncture sites. There is no personal or family history of a bleeding tendency. Haemoglobin, 9.9; white cell count, 6.3; platelets, 300; prothrombin time 26 s, activated partial thromboplastin time 60 s, thrombin time 18 s, bleeding time 5 min. B - Disseminated intravascular coagulation (DIC)« YOUR ANSWER K - Vitamin K deficiency« CORRECT ANSWER. A post-operative vitamin K deficiency is not an uncommon haemostatic defect, especially in this particular scenario. Vitamin K is a fat-soluble vitamin absorbed in the terminal ileum with bile salts. Prolonged diarrhoea/high ileostomy output may result in abnormalities of vitamin K absorption. This, combined with treatment with broad-spectrum antibiotics, results in suppression of the normal gut bacterial flora that are in turn an important producer of vitamin K. This condition is best managed with slow intravenous infusion of vitamin K and treatment of the underlying cause of deficiency. Scenario 3 A male infant suffers from a profuse post-circumcision haemorrhage and his mother mentions that her brother had a clotting problem that requires regular Factor VIII infusions. C - Factor V Leiden deficiency« YOUR ANSWER D - Haemophilia A« CORRECT ANSWER. Haemophilia is the commonest hereditary disorder of blood coagulation. The inheritance is sex-linked but 33% of patients have no family history, and presumably result from spontaneous mutations. Haemophilia A is an absence, or low level, of plasma factor VIII, while Haemophilia B (Christmas disease) is the result of a deficiency in factor IX. Clinically the conditions are identical and can only be distinguished on coagulation factor assays. The above presentation is common while recurrent painful haemarthroses and muscle haematomas dominate the clinical picture of severely affected individuals, resulting in progressive deformity. Bleeding episodes are treated with factor VIII replacement or desmopressin. For major surgery, levels are raised to 100% and maintained at 60% until bleeding has ceased. 04. Theme: Bowel preparation A No specific preparation B Clear fluids for 24 h prior to procedure C Phosphate enema prior to procedure D Sodium picosulphate (1 sachet bd) and clear fluids for 24 h prior to procedure. From the list above, choose the most appropriate bowel preparation regimen for each of the following situations. Each answer may be used once, several times or not at all. Scenario 1

Page 4: Page | 1104

MyPasTest » MRCS A Online - Jan Exam 2015 14. Principles of Surgery in General; Perioperative Care (55Qs)

-------------------------------------------------------------------------------------------------------------------

---------------------------------------------------------------------------------------------------------------------------------------------------------------------------------------------------------------

Dr Mohammed Shamsul Islam Khan, Medical Officer, Clinical Neuro-Surgery, National Institute of Neuro-Sciences and Hospital

Sher-E-Bangla Nagar, Dhaka-1207, Bangladesh. Mobile: +880 1713 455 662, +880 1685 811979. E-mail: [email protected]

Page |

1107

A 33-year-old male awaiting elective terminal ileal resection for a small bowel stricture resulting from Crohn’s disease. A - No specific preparation« YOUR ANSWER B - Clear fluids for 24 h prior to procedure« CORRECT ANSWER. Preparing the small bowel is difficult; 24 h of clear fluids should help keep the content low residue. Scenario 2 A 46-year-old female attending for a colonoscopy to investigate a change in bowel habit. B - Clear fluids for 24 h prior to procedure« YOUR ANSWER D - Sodium picosulphate (1 sachet bd) and clear fluids for 24 h prior to procedure« CORRECT ANSWER. Where possible the large bowel should be fully prepared by 24 h of clear fluids and purgative agents such as sodium picosulphate. This maximises emptying of the bowel and optimises the views obtained at colonoscopy. Scenario 3 A 28-year-old male who attends for a flexible sigmoidoscopy to investigate per rectal bleeding. C - Phosphate enema prior to procedure« CORRECT ANSWER. A phosphate enema causes emptying of the distal bowel and maximises views at flexible sigmoidoscopy. Scenario 4 A 73-year-old male awaiting a low anterior resection for an obstructing rectal carcinoma. D - Sodium picosulphate (1 sachet bd) and clear fluids for 24 h prior to procedure« YOUR ANSWER B - Clear fluids for 24 h prior to procedure« CORRECT ANSWER. Ideally this patient should have full bowel preparation with sodium picosulphate. However, in the case of obstructing tumours this is clearly hazardous. The patient should maintain a period of 24 h of clear fluids and should already be on a low residue diet. Scenario 5 A 19-year-old male who has acute appendicitis and is scheduled for an appendicectomy. E - « YOUR ANSWER A - No specific preparation« CORRECT ANSWER. No preparation, other than standard nil by mouth for 6 h prior to procedure, is required for patients undergoing an appendicectomy. 05. Theme: Operative management A Cancel surgery B Carry on with surgery regardless C Conservative management D Immediate surgery and intensive care unit booking E Stop pill and w ait 4 weeks F Wait 6 months.

Page 5: Page | 1104

MyPasTest » MRCS A Online - Jan Exam 2015 14. Principles of Surgery in General; Perioperative Care (55Qs)

-------------------------------------------------------------------------------------------------------------------

---------------------------------------------------------------------------------------------------------------------------------------------------------------------------------------------------------------

Dr Mohammed Shamsul Islam Khan, Medical Officer, Clinical Neuro-Surgery, National Institute of Neuro-Sciences and Hospital

Sher-E-Bangla Nagar, Dhaka-1207, Bangladesh. Mobile: +880 1713 455 662, +880 1685 811979. E-mail: [email protected]

Page |

1108

For each of the clinical scenarios described below, select the most appropriate management plan from the above list. Each option may be used once, more than once, or not at all. Scenario 1 A 25-year-old woman is diagnosed with acute appendicitis at the beginning of the third trimester of her pregnancy A - Cancel surgery« YOUR ANSWER B - Carry on with surgery regardless« CORRECT ANSWER. Scenario 2 A 25-year-old woman is diagnosed with acute appendicitis and is taking the oral contraceptive pill B - Carry on with surgery regardless« CORRECT ANSWER. Scenario 3 A 30-year-old woman is on your waiting list for a right inguinal hernia repair but is on the oral contraceptive pill C - Conservative management« YOUR ANSWER E - Stop pill and w ait 4 weeks« CORRECT ANSWER. Appendicitis in pregnancy may be difficult to diagnose due to upward displacement of the appendix and masking of guarding by the uterus. The later into the pregnancy, the greater is the mortality risk to the mother and fetus. A patient with acute appendicitis who is on the oral contraceptive pill should have an emergency operation. Neither pregnancy, nor taking the oral contraceptive pill is a contraindication to appendicectomy. The correct procedure for women on the oral contraceptive pill is to carry on with the surgery but take precautionary measures, such as thromboembolic deterrent stockings (TEDS), pneumatic calf compression and administration of heparin or low-molecular-weight heparins. A patient requiring elective right inguinal hernia repair taking the oral contraceptive pill should have the surgery delayed, as the risk of potential complications (ie PE) would outweigh waiting for 1 month and then performing the surgery. During this 1-month delay, the patient should be advised to stop taking the oral contraceptive pill and to use other forms of contraception. 06. THEME: Appropriate imaging A Request a repeat abdominal film at 24 h and review B Request an abdominal ultrasound C Request a pelvic ultrasound D Request a computed tomography (CT) scan E Request portable bedside ultrasound. For each of the following situations, select the most likely answer from the above list. Each option may be used once, more than once or not at all. Scenario 1 A 14-year-old girl attends the Accident and Emergency Department with her concerned mother. She complains of a 12-h history of worsening low abdominal pain and light headedness. Her b-human chorionic gonadotropin (b-HCG) test is positive but she asks you not to tell her mother. She is not keen to stay in hospital. A - Request a repeat abdominal film at 24 h and review« YOUR ANSWER C - Request a pelvic ultrasound« CORRECT ANSWER.

Page 6: Page | 1104

MyPasTest » MRCS A Online - Jan Exam 2015 14. Principles of Surgery in General; Perioperative Care (55Qs)

-------------------------------------------------------------------------------------------------------------------

---------------------------------------------------------------------------------------------------------------------------------------------------------------------------------------------------------------

Dr Mohammed Shamsul Islam Khan, Medical Officer, Clinical Neuro-Surgery, National Institute of Neuro-Sciences and Hospital

Sher-E-Bangla Nagar, Dhaka-1207, Bangladesh. Mobile: +880 1713 455 662, +880 1685 811979. E-mail: [email protected]

Page |

1109

Regardless of the girls desire to leave, she has an ectopic pregnancy that should be confirmed by an emergency pelvic ultrasound and once seen, surgery should be performed without delay. She must not leave the hospital as she is at very high risk of fatal pelvic haemorrhage and death. Scenario 2 A 65-year-old man attends the surgical receiving ward with altered bowel habit, epigastric pain and abdominal distension dating back over the previous week. He is cardiovascularly stable and afebrile. His abdominal film shows a notably distended transverse colon with thumb printing. B - Request an abdominal ultrasound« YOUR ANSWER A - Request a repeat abdominal film at 24 h and review« CORRECT ANSWER. The man is stable and the diagnosis of toxic megacolon is therefore not made although this must be borne in mind. He may respond well to initial conservative measurements with steroid preparations and series abdominal films on a daily basis to monitor the progress of this are commonplace. The man may eventually go to surgery but the outcome will be improved if this can be done on an elective rather than emergency basis. Scenario 3 A 68-year-old man attends the Accident and Emergency Department with a history of severe back pain within the last hour and feeling ‘dizzy’. He is moving in and out of responsiveness and has a blood pressure (BP) of 90/40. C - Request a pelvic ultrasound« YOUR ANSWER E - Request portable bedside ultrasound« CORRECT ANSWER. This man is profoundly shocked and has a ruptured abdominal aortic aneurysm. His only chance of survival is to get to theatre as soon as possible. The diagnosis should be confirmed by an emergency portable ultrasound scan of his abdomen by the bedside. Scenario 4 A 45-year-old woman with right upper quadrant pain, nausea and vomiting dating back over the last week. She also complains of marked dyspepsia. D - Request a computed tomography (CT) scan« YOUR ANSWER B - Request an abdominal ultrasound« CORRECT ANSWER. This common presentation will require an abdominal ultrasound while fasted to look for gallstones as well as other hepatobiliary disorders. If negative an upper gastrointestinal endoscopy would be appropriate. 07. Theme: Pre-operative anaesthetic assessment A American Society of Anesthesiologists' score (ASA) I B ASA II C ASA III D ASA IV E ASA V. From the list above, choose the most appropriate level of anaesthetic assessment for each of the following scenarios. Each answer may be used once, several times or not at all. Scenario 1

Page 7: Page | 1104

MyPasTest » MRCS A Online - Jan Exam 2015 14. Principles of Surgery in General; Perioperative Care (55Qs)

-------------------------------------------------------------------------------------------------------------------

---------------------------------------------------------------------------------------------------------------------------------------------------------------------------------------------------------------

Dr Mohammed Shamsul Islam Khan, Medical Officer, Clinical Neuro-Surgery, National Institute of Neuro-Sciences and Hospital

Sher-E-Bangla Nagar, Dhaka-1207, Bangladesh. Mobile: +880 1713 455 662, +880 1685 811979. E-mail: [email protected]

Page |

1110

A 35-year-old female who attends for a laparoscopic cholecystectomy. She has a background of controlled essential hypertension for which she takes atenolol. A - ASA I« YOUR ANSWER B - ASA II« CORRECT ANSWER. This patient is due to have an intermediate procedure. With her background of essential hypertension her American Society of Anesthesiologists’ (ASA) grade is II as this is a mild systemic disease. Scenario 2 A 78-year-old male who is scheduled for a right hemicolectomy for a caecal tumour. He has a background of a previous myocardial infarction and angina. He develops chest pain after walking 50 m. B - ASA II« YOUR ANSWER. This patient is due for a major procedure. With his background of ischaemic heart disease and chest pain on mild exertion he is a high anaesthetic risk. His ASA grade is IV. Scenario 3 A 17-year-old male presents with right iliac fossa pain and vomiting. He has acute appendicitis and is scheduled for an appendicectomy. He is otherwise fit and well. C - ASA III« YOUR ANSWER A - ASA I« CORRECT ANSWER. ASA I This patient is young and in good health. There are no co-existing medical issues and hence the ASA grade is I. YOUR ANSWER WAS INCORRECT Scenario 4 A 75-year-old male presents to the emergency department with severe abdominal pain radiating through to his back. He has a ruptured abdominal aortic aneurysm. He has ischaemic heart disease and hypertension D - ASA IV« YOUR ANSWER E - ASA V« CORRECT ANSWER. This patient has a background of ischaemic heart disease and hypertension. A leaking abdominal aortic aneurysm can only be dealt with by surgical intervention. If he does not undergo an operation he will not survive more than 24 h. Scenario 5 A 58-year-old female presents with a left femoral hernia. She has chronic obstructive pulmonary disease (COPD) and becomes breathless on mild exertion, eg climbing a flight of stairs. E - ASA V« YOUR ANSWER C - ASA III« CORRECT ANSWER. This patient has a systemic disease that limits activity. Although significant, the chronic obstructive pulmonary disease (COPD) is not immediately life threatening, hence the ASA grade is III. 08. Theme: Acid-base balance disturbances pH p(CO2) HCO3-

A 7.20 3.1 11

Page 8: Page | 1104

MyPasTest » MRCS A Online - Jan Exam 2015 14. Principles of Surgery in General; Perioperative Care (55Qs)

-------------------------------------------------------------------------------------------------------------------

---------------------------------------------------------------------------------------------------------------------------------------------------------------------------------------------------------------

Dr Mohammed Shamsul Islam Khan, Medical Officer, Clinical Neuro-Surgery, National Institute of Neuro-Sciences and Hospital

Sher-E-Bangla Nagar, Dhaka-1207, Bangladesh. Mobile: +880 1713 455 662, +880 1685 811979. E-mail: [email protected]

Page |

1111

B 7.42 5.3 26 C 7.56 3.0 32 For each of the scenarios given match the most appropriate answer. Each option may be used once, more than once, or not at all. Scenario 1 A 68-year-old woman with small bowel obstruction and vomiting for 72 hours, who exhibited signs of peritonitis and a white blood cell count (WBC) of 23 ´ 109/litre. A - 7.20 3.1 11« CORRECT ANSWER. Intestinal obstruction will lead to loss of fluid from the intravascular space and cause loss of the third space. Scenario 2 A 45-year-old woman who had a total abdominal hysterectomy for fibroids is now complaining of shortness of breath 7 days after surgery. There is no history of haemoptysis or pleuritic chest pain. B - 7.42 5.3 26« YOUR ANSWER C - 7.56 3.0 32« CORRECT ANSWER. It is likely that, with the presence of peritonism and a grossly elevated WBC, infarcted bowel should be strongly suspected, thereby producing a metabolic acidosis. Scenario 3 A 75-year-old man who had an uneventful abdominal aortic aneurysm repair 10 days ago, and is known to smoke 30 cigarettes a day. C - 7.56 3.0 32« YOUR ANSWER B - 7.42 5.3 26« CORRECT ANSWER. Pulmonary emboli may occur at any stage after surgery, most frequently after 5 days and should be suspected in all cases of dyspnoea (earliest sign. Intestinal obstruction will lead to loss of fluid from the intravascular space and cause loss of the third space. It is likely that, with the presence of peritonism and a grossly elevated WBC, infarcted bowel should be strongly suspected, thereby producing a metabolic acidosis. Pulmonary emboli may occur at any stage after surgery, most frequently after 5 days and should be suspected in all cases of dyspnoea (earliest sign). 09. Theme: Acid–base balance/status A Acute renal failure B Flail chest C Pulmonary embolus D Pyloric stenosis. For each of the acid–base disturbance conditions given below, select the most likely diagnosis from the above list. Each option may be used once, more than once, or not at all. Scenario 1 Hypocapnia A - Acute renal failure« YOUR ANSWER

Page 9: Page | 1104

MyPasTest » MRCS A Online - Jan Exam 2015 14. Principles of Surgery in General; Perioperative Care (55Qs)

-------------------------------------------------------------------------------------------------------------------

---------------------------------------------------------------------------------------------------------------------------------------------------------------------------------------------------------------

Dr Mohammed Shamsul Islam Khan, Medical Officer, Clinical Neuro-Surgery, National Institute of Neuro-Sciences and Hospital

Sher-E-Bangla Nagar, Dhaka-1207, Bangladesh. Mobile: +880 1713 455 662, +880 1685 811979. E-mail: [email protected]

Page |

1112

C - Pulmonary embolus« CORRECT ANSWER. Pulmonary embolus (small or medium sized) usually produces a lowered PCO2 because of increased respiratory rate. Scenario 2 Metabolic acidosis B - Flail chest« YOUR ANSWER A - Acute renal failure« CORRECT ANSWER. Scenario 3 Metabolic alkalosis C - Pulmonary embolus« YOUR ANSWER D - Pyloric stenosis« CORRECT ANSWER. Vomiting due to pyloric stenosis causes volume depletion and loss of H+. The obstruction between the stomach and the duodenum also results in a decreased loss of HCO3

–. A resultant metabolic alkalosis may occur if duodenal secretion continues and renal excretion of HCO3

– is insufficient to correct for the plasma rise in HCO3

–. A flail chest occurs when a segment of the chest wall has no bony continuity with the rest of the thoracic cage. This usually results from trauma associated with multiple rib fractures. Underlying lung injury and restricted chest wall movement will contribute to the patient’s hypoxia. 10. Theme: Nutrition A High calorie and protein supplements, eg Fortijuice, Fortisip B Nasogastric (NG) feeding C Perenteral gastrostomy tube (PEG) D Feeding jejunostomy E Total parenteral nutrition (TPN) From the list above, choose the most appropriate mode of delivering nutrition for each of the following situations. Each answer may be used once, several times or not at all. Scenario 1 A 71-year-old man who is scheduled for a total gastrectomy for a gastric carcinoma. He is unable to tolerate oral diet due to projectile vomiting immediately after eating. He has a body mass index (BMI) of 19 and has an albumin of 27 g/l. A - High calorie and protein supplements, eg Fortijuice, Fortisip« YOUR ANSWER E - Total parenteral nutrition (TPN)« CORRECT ANSWER. This patient is pre-operative and needs to be optimised prior to surgery. He has a low body mass index (BMI) and albumin level. The projectile vomiting suggests acquired pyloric stenosis due to an obstructing tumour. This excludes the use of the oral and nasogastric route. He should be given TPN either peripherally, or preferably centrally. Scenario 2 A 68-year-old female who one week post-appendicectomy. She is making a good recovery, tolerating diet and fluids, and her bowel is functioning. Her appetite is poor and her albumin is 28 g/l. B - Nasogastric (NG) feeding« YOUR ANSWER

Page 10: Page | 1104

MyPasTest » MRCS A Online - Jan Exam 2015 14. Principles of Surgery in General; Perioperative Care (55Qs)

-------------------------------------------------------------------------------------------------------------------

---------------------------------------------------------------------------------------------------------------------------------------------------------------------------------------------------------------

Dr Mohammed Shamsul Islam Khan, Medical Officer, Clinical Neuro-Surgery, National Institute of Neuro-Sciences and Hospital

Sher-E-Bangla Nagar, Dhaka-1207, Bangladesh. Mobile: +880 1713 455 662, +880 1685 811979. E-mail: [email protected]

Page |

1113

A - High calorie and protein supplements, eg Fortijuice, Fortisip« CORRECT ANSWER. This patient is making a slow recovery from an appendicectomy. This is common in more elderly patients. As she is able to tolerate diet and fluids, high calorie and protein supplements should be introduced. Scenario 3 A 35-year-old male who presents 2 weeks following a small bowel resection for Crohn’s disease. He has developed an enterocutaneous fistula that has an output of 450 ml per day. His is able to tolerate diet and fluids and his albumin level is 18 g/l. C - Perenteral gastrostomy tube (PEG)« YOUR ANSWER E - Total parenteral nutrition (TPN)« CORRECT ANSWER. In this patient TPN should be introduced to optimise his condition. TPN forms part of the ‘SNAP’ method of managing a fistula. S is for control of sepsis, N is for nutrition in the form of TPN, A is for anatomy, define the anatomy by the use of contrast radiology, and P is for plan, as in form a management plan. An entero-cutaneous fistula may close spontaneously if managed appropriately. Scenario 4 A 25-year-old female who has undergone a subtotal colectomy for ulcerative colitis. She is 10 days post-procedure and making a steady recovery. Her appetite remains poor and her albumin level is 23 g/l. D - Feeding jejunostomy« YOUR ANSWER B - Nasogastric (NG) feeding« CORRECT ANSWER. This patient is 10 days post-procedure and making a good recovery. Her albumin level is low along with poor appetite. There is a role for high calorie and protein supplementation via the oral route, but this is likely to be poorly tolerated. NG feeding would deliver suitable nutrition via the enteral route without the need for TPN. 11. Theme: Shock A Cardiogenic shock B Fat embolism C Hypovolaemic shock D Thromboembolism. For each of the scenarios listed below, select the most likely diagnosis from the above list. Each option may be used once, more than once, or not at all. Scenario 1 A 26-year-old man with a comminuted closed fracture of the femur shaft undergoes intramedullary nail fixation. Two days postoperatively, he develops a pyrexia, shortness of breath and a tachycardia. A - Cardiogenic shock« YOUR ANSWER B - Fat embolism« CORRECT ANSWER. Fat embolism occurs in patients with multiple closed fractures, but has been reported in those with other skeletal trauma and burns. Fat embolism causes a pyrexia, tachycardia, shortness of breath, confusion and petechial haemorrhages, especially on the chest and conjunctivae.

Page 11: Page | 1104

MyPasTest » MRCS A Online - Jan Exam 2015 14. Principles of Surgery in General; Perioperative Care (55Qs)

-------------------------------------------------------------------------------------------------------------------

---------------------------------------------------------------------------------------------------------------------------------------------------------------------------------------------------------------

Dr Mohammed Shamsul Islam Khan, Medical Officer, Clinical Neuro-Surgery, National Institute of Neuro-Sciences and Hospital

Sher-E-Bangla Nagar, Dhaka-1207, Bangladesh. Mobile: +880 1713 455 662, +880 1685 811979. E-mail: [email protected]

Page |

1114

Scenario 2 A 72-year-old man with an underlying prostate carcinoma sustains a femoral shaft fracture. He undergoes intramedullary nail fixation. At post-operative day 7, he develops shortness of breath, hypotension and a tachycardia. B - Fat embolism« YOUR ANSWER D - Thromboembolism« CORRECT ANSWER. Post-operative day 7 is the characteristic time for a thromboembolism. Scenario 3 A 60-year-old man develops sudden back pain and is brought to the Emergency Department with a swollen, tense abdomen. He is tachycardic, with a low-volume pulse and low BP. C - Hypovolaemic shock« CORRECT ANSWER. Scenario 3 describes a leaking abdominal aortic aneurysm (AAA). Management involves immediate transfer to the operating theatre for laparotomy and emergency grafting of his AAA. 12. Theme: Types of anaemia A Aplastic anaemia B Autoimmune haemolytic anaemia C Chronic disease D Folate deficiency E Haemorrhage F Hereditary spherocytosis G Hypersplenism H Iron deficiency I Microangiopathic haemolytic anaemia J Pernicious anaemia K Sickle cell anaemia L Sideroblastic anaemia M Thalassaemia N Vitamin B12 deficiency. The following are descriptions of patients’ symptoms with types of surgically important anaemia. Please select the most appropriate diagnosis from the list. The items may be used once, more than once, or not at all. Scenario 1 A 47-year-old Afro-Caribbean man presents with fatigue, diarrhoea and steatorrhoea 12-weeks after ileal resection for isolated Crohn’s disease. Examination is unremarkable. Full blood count reveals haemoglobin 8.6 g/dl and mean corpuscular volume 110 fl. A - Aplastic anaemia« YOUR ANSWER N - Vitamin B12 deficiency« CORRECT ANSWER. This patient’s blood test reveals a macrocytic anaemia. This could have megaloblastic (vitamin B12 or folate deficiency) or non-megaloblastic(alcohol, liver disease) causes. Complete resection of the terminal ileum results in the malabsorption of vitamin B12 in association with intrinsic factor and bile salts, consequently leading to megaloblastic anaemia,steatorrhoea and diarrhoea. The Schilling test

Page 12: Page | 1104

MyPasTest » MRCS A Online - Jan Exam 2015 14. Principles of Surgery in General; Perioperative Care (55Qs)

-------------------------------------------------------------------------------------------------------------------

---------------------------------------------------------------------------------------------------------------------------------------------------------------------------------------------------------------

Dr Mohammed Shamsul Islam Khan, Medical Officer, Clinical Neuro-Surgery, National Institute of Neuro-Sciences and Hospital

Sher-E-Bangla Nagar, Dhaka-1207, Bangladesh. Mobile: +880 1713 455 662, +880 1685 811979. E-mail: [email protected]

Page |

1115

distinguishes between malabsorption of vitamin B12 and lack of intrinsic factor. (A megaloblast is a cell in which cytoplasmic and nuclear maturation are out of phase because nuclear maturation is slow. Scenario 2 A 42-year-old Caucasian woman presents to The Emergency Department with jaundice and right upper quadrant abdominal pain; her past medical history includes systemic lupus erythematosis. Examination confirms clinical jaundice and reveals moderate splenomegaly. Blood tests reveal haemoglobin 9.0 g/dl and mean corpuscular volume 90 fl. She has a positive direct Coombs’ test. B - Autoimmune haemolytic anaemia« CORRECT ANSWER. The blood test reveals a normocytic anaemia of which one of the causes ishaemolysis. Haemolysis may result in unconjugated hyperbilirubinaemia, hence jaundice and gallstones, and splenic hypertrophy. Causes are either hereditary or acquired. Acquired haemolysis may be either immune (e.g. autoimmune – cold or warm antibody-mediated) or non-immune (e.g. microangiopathic). The direct Coombs’ test identifies cells coated with antibody or complement, a positive result indicating an immune cause of haemolysis. Warm autoimmune haemolytic anaemia is associated with other autoimmune disorders, especially systemic lupus erythematosis. Scenario 3 A 74-year-old Afro-Caribbean man presents with lethargy, weightloss and right-sided abdominal pain. Examination is unremarkable.Blood tests reveal haemoglobin 10.0 g/dl and mean corpuscularvolume 72 fl. C - Chronic disease« YOUR ANSWER H - Iron deficiency« CORRECT ANSWER. The blood test reveals a microcytic anaemia. One of the commonest causes of which is iron deficiency anaemia, secondary to chronic blood loss. This patient’s history should immediately alert you to the possibility of right-sided colonic carcinoma, which frequently results in faecal occult blood loss. 13. Theme: Consent A The Patient B Parent or legal guardian C Other relative D Consultant E Two junior doctors. From the list above, choose the most appropriate person or persons who can give informed consent for the following scenarios. Each answer may be used once, several times or not at all. Scenario 1 A 46-year-old female with learning difficulties attends the emergency department with heavy fresh rectal bleeding. The haemoglobin level is 9.5 g/dl. As the duty senior house officer on call you arrange for a flexible sigmoidoscopy to be performed. You discuss the procedure and are assured that the patient is able comprehend the indications for the procedure. The patient refuses the procedure. Can consent be obtained and from who? A - The Patient« CORRECT ANSWER.

Page 13: Page | 1104

MyPasTest » MRCS A Online - Jan Exam 2015 14. Principles of Surgery in General; Perioperative Care (55Qs)

-------------------------------------------------------------------------------------------------------------------

---------------------------------------------------------------------------------------------------------------------------------------------------------------------------------------------------------------

Dr Mohammed Shamsul Islam Khan, Medical Officer, Clinical Neuro-Surgery, National Institute of Neuro-Sciences and Hospital

Sher-E-Bangla Nagar, Dhaka-1207, Bangladesh. Mobile: +880 1713 455 662, +880 1685 811979. E-mail: [email protected]

Page |

1116

In this case the patient is refusing to give consent for further investigation. If the patient is deemed competent in the decision-making process then their wishes should be respected. Competence is a difficult area. Scenario 2 A 16-year-old female presents with one day of right iliac fossa pain and vomiting. The white cell count is 15 × 109/l and the C-reactive protein level is 58g/l. A diagnosis of acute appendicitis is formed and you make the necessary arrangements for theatre. The patient is present with her grandmother as her parents are away and uncontactable. Both she and the Grandmother have had the procedure explained fully and you are satisfied they understand the information.Who can give consent? B - Parent or legal guardian« YOUR ANSWER. As the patient is 16 years old, she is old enough to give consent herself. For patients under 16 years of age the usual method is to obtain consent from the parent. However, in an acute situation, if the attending doctor deems the patient competent in the decision-making process, consent may be obtained. Every effort should be made to discuss this with the parent where possible. Scenario 3 A patient involved in a road traffic accident is brought unconscious into the emergency department. He has multiple injuries including significant intra-abdominal haemorrhage confirmed on a computerised tomography (CT). He requires an emergency laparotomy to control the bleeding. He is unaccompanied. Who can give consent? C - Other relative« YOUR ANSWER D - Consultant« CORRECT ANSWER. In this circumstance the emphasis is on emergency care of the patient. If an acute situation presents where the patient requires definitive treatment, time should not be lost. A senior doctor (ideally the consultant in charge of the patient’s care) can act in the patient’s best interest. Scenario 4 A 78-year-old man attends the emergency department with sudden onset severe abdominal pain radiating through to his back. He is tachycardic and hypotensive. He is reviewed by the on-call registrar who diagnoses a leaking abdominal aortic aneurysm. His Glasgow Coma Score is 11 and he is unable to comprehend the implications of surgery and consent. His wife says that she will sign the consent form.In this instance who can give consent? D - Consultant« CORRECT ANSWER. When a patient clearly needs emergency surgery and the patient cannot consent for themselves, a senior doctor (ideally the consultant in charge of the patient’s care) can sign a consent form in order to proceed with appropriate treatment. The patient’s wife cannot sign the consent form on behalf of the patient. Most consent forms now have a section where the relative can sign to say they are in agreement with the management advised by the attending doctor(s). Scenario 5 An 8-year-old boy attends for an elective repair of a right inguinal hernia. As the senior house officer you seek consent. Who can give consent? E - Two junior doctors« YOUR ANSWER B - Parent or legal guardian« CORRECT ANSWER.

Page 14: Page | 1104

MyPasTest » MRCS A Online - Jan Exam 2015 14. Principles of Surgery in General; Perioperative Care (55Qs)

-------------------------------------------------------------------------------------------------------------------

---------------------------------------------------------------------------------------------------------------------------------------------------------------------------------------------------------------

Dr Mohammed Shamsul Islam Khan, Medical Officer, Clinical Neuro-Surgery, National Institute of Neuro-Sciences and Hospital

Sher-E-Bangla Nagar, Dhaka-1207, Bangladesh. Mobile: +880 1713 455 662, +880 1685 811979. E-mail: [email protected]

Page |

1117

In this situation where the child is under 16 years of age, appropriate consent should be obtained from the parent. 14. A 56-year-old man presents in clinic requesting repair of an abdominal hernia. He has had an myocardial infarct 6 weeks ago. What is his likely risk of post-operative re-infarction?

0-10%« YOUR ANSWER

11-20%

21-30%

31-40%« CORRECT ANSWER

41-50%. The risk of further stress on cardiac function is increased with a GA. A large study by Tarhan et al showed that an operation within 3 months following a myocardial infarct was associated with a 37% risk of re-infarction which reduced to 16% at 3-6 months and then to 4-5% after 6 months. Thus any elective surgery should be avoided for a period of six months. 15. A 56-year-old man with angina who takes GTN spray occasionally presents at the pre-assessment clinic prior to an elective cholecystectomy. What ASA grade is he?

ASA I« YOUR ANSWER

ASA II« CORRECT ANSWER

ASA III

ASA IV

ASA V. The American society of Anaesthesiologists (ASA) grade predicts a patients mortality based on medical co-morbidities.

Definition Mortality %

I Normal healthy individual 0.06

II Mild systemic disease that does not limit activity 0.4

III Severe systemic disease that limits activity but is not incapacitating 4.5

IV Incapacitating systemic disease which is constantly life threatening 23

V Moribund, not expected to survive 24 hours with or without surgery 51

16. Theme: General anaesthetic agents A Atracurium B Dantrolene sodium C Enflurane D Etomidate E Glycopyrronium

Page 15: Page | 1104

MyPasTest » MRCS A Online - Jan Exam 2015 14. Principles of Surgery in General; Perioperative Care (55Qs)

-------------------------------------------------------------------------------------------------------------------

---------------------------------------------------------------------------------------------------------------------------------------------------------------------------------------------------------------

Dr Mohammed Shamsul Islam Khan, Medical Officer, Clinical Neuro-Surgery, National Institute of Neuro-Sciences and Hospital

Sher-E-Bangla Nagar, Dhaka-1207, Bangladesh. Mobile: +880 1713 455 662, +880 1685 811979. E-mail: [email protected]

Page |

1118

F Halothane G Isoflurane H Neostigmine I Nitrous oxide J Propofol K Suxamethonium L Thiopentone sodium. The above are all types of general anaesthetic agents. Indicate the most likely drug from the list that fits the clinical scenarios below. The items may be used once, more than once, or not at all. Scenario 1 A 29-year-old man, involved in a serious road traffic accident, requires an urgent laparotomy. Rapid sequence induction is undertaken by the anaesthetic Senior House Officer. It is noted postoperatively that the patient appears to take longer than usual to begin self-ventilating. Which drug would be responsible for this? A - Atracurium « YOUR ANSWER K - Suxamethonium« CORRECT ANSWER. Suxamethonium is a depolarising muscle relaxant that mimics the action of acetylcholine. In this case it was chosen for its rapid onset (as would be required for crash induction). The delay in self-ventilation is known as suxamethonium, or scoline, apnoea. In a normal individual the effects of this agent last for only a few minutes until it has been metabolised by plasma cholinesterase. Deficiency of this enzyme leads to prolonged action, causing apnoea. Scenario 2 A 35-year-old woman undergoing elective laparoscopy is induced using an intravenous anaesthetic agent. Her systolic blood pressure rapidly drops to 70 mmHg and a wheeze is audible in the anaesthetic room. Which drug would have been given? B - Dantrolene sodium « YOUR ANSWER L - Thiopentone sodium« CORRECT ANSWER. This is the oldest, cheapest and best understood intravenous induction agent. However, it has been superseded by propofol and etomidate, because of its potential side-effects of marked hypotension, anaphylaxis and bronchospasm. It can also leave patients with a bad hangover. Etomidate is less of a myocardial depressant and is used in patients who have cardiovascular instability. Propofol is the most widely used intravenous agent, allowing for a clear-headed and rapid recovery. It should be used with caution in hypovolaemic patients. Scenario 3 A previously well, 50-year-old woman is noted in the post-operative period to develop progressive jaundice. She was maintained on an inhalational anaesthetic agent for an open reduction and internal fixation of an ankle fracture. Liver function tests: bilirubin, 190; aspartate aminotransferase (AST), 345; alanine aminotransferase (ALP), 120; γ -glutamyltransferase, 115. Which drug would fit with this subsequent scenario? C - Enflurane « YOUR ANSWER F - Halothane « CORRECT ANSWER.

Page 16: Page | 1104

MyPasTest » MRCS A Online - Jan Exam 2015 14. Principles of Surgery in General; Perioperative Care (55Qs)

-------------------------------------------------------------------------------------------------------------------

---------------------------------------------------------------------------------------------------------------------------------------------------------------------------------------------------------------

Dr Mohammed Shamsul Islam Khan, Medical Officer, Clinical Neuro-Surgery, National Institute of Neuro-Sciences and Hospital

Sher-E-Bangla Nagar, Dhaka-1207, Bangladesh. Mobile: +880 1713 455 662, +880 1685 811979. E-mail: [email protected]

Page |

1119

Halothane is a volatile, inhalational anaesthetic, which is well tolerated and non-irritant. It can cause respiratory depression and acts as a negative inotrope. Its major disadvantage, as highlighted by this case, is severe hepatotoxicity (1 in 30,000), with 50% mortality. This is caused when its oxidative metabolites induce immune-mediated hepatic necrosis. As with all inhalational anaesthetics (apart from nitrous oxide), it can be responsible for malignant hyperthermia (1 in 150,000), an inherited autosomal dominant condition. This is therapeutically managed using intravenous dantrolene sodium. Scenario 4 An extreme sportsman who has recently returned from a diving trip to Egypt is awaiting elective repair of his right inguinal hernia. What agent should be avoided in his general anaesthetic regimen? D - Etomidate « YOUR ANSWER I - Nitrous oxide« CORRECT ANSWER. Nitrous oxide is a potent analgesic, but only a weak anaesthetic. It is used to potentiate the action of other inhalational anaesthetics, allowing a reduction in the dose required. Nitrous oxide will diffuse into any aircontaining space, and does so more rapidly than nitrogen. It must not be used in patients who have recently been diving or who have been exposed to high atmospheric pressures, or in patients suspected of having a gas- filled space (eg pneumothorax). 17. An 85-year-old lady who is unfit for a GA undergoes spinal anaesthesia for debridement of a haematoma. Which of the following is a common complication of spinal anaesthesia?

Epidural haematoma« YOUR ANSWER

Hypotension« CORRECT ANSWER

Infection

Meningism

Toxicity. In a spinal anaesthetic, local anaesthesia is placed into CSF below the termination of cord at L1, whereas in an epidural the local anaesthetic is placed into the fatty epidural place which is more challenging to perform and inherently more complications arise. Although infection, toxicity and meningism can occur, hypotention is a much more common complication. 18. THEME: Blood gases A Metabolic acidosis B Metabolic alkalosis C Respiratory acidosis D Respiratory alkalosis E Chronic respiratory acidosis F Normal acid/base balance. For each of the scenarios match the most appropriate cause. Each option may be used once, more than once, or not at all. Scenario 1 A patient presents with COPD.

Page 17: Page | 1104

MyPasTest » MRCS A Online - Jan Exam 2015 14. Principles of Surgery in General; Perioperative Care (55Qs)

-------------------------------------------------------------------------------------------------------------------

---------------------------------------------------------------------------------------------------------------------------------------------------------------------------------------------------------------

Dr Mohammed Shamsul Islam Khan, Medical Officer, Clinical Neuro-Surgery, National Institute of Neuro-Sciences and Hospital

Sher-E-Bangla Nagar, Dhaka-1207, Bangladesh. Mobile: +880 1713 455 662, +880 1685 811979. E-mail: [email protected]

Page |

1120

A - Metabolic acidosis« YOUR ANSWER E - Chronic respiratory acidosis« CORRECT ANSWER. A patient with COPD would have a chronic respiratory acidosis. Scenario 2 A patient presents with pH 7.6, pO2 10.2 kPa, pCO2 5.8 kPa, HCO3

- 32 mmol/litre. B - Metabolic alkalosis« CORRECT ANSWER. The patient in this clinical scenario has alkalosis, as indicated by the pH of 7.6. This is metabolic as the bicarbonate is 32mmol/litre. Scenario 3 A patient presents with pH 7.3, pO28 kPa, pCO2 8.2 kPa, HCO3

- 28 mmol/litre. C - Respiratory acidosis« CORRECT ANSWER The patient in this clinical scenario has respiratory acidosis, as indicated by a pH of 7.3 and pCO2 of 8.2. The bicarbonate here is normal. Scenario 4 A patient presents with pH 7.2, pO2 14 kPa, pCO2 3.5 kPa, HCO3

- 12 mmol/litre. D - Respiratory alkalosis« YOUR ANSWER A - Metabolic acidosis« CORRECT ANSWER. Low HCO3, low pH and low pCO2 represent a picture of metabolic acidosis. Scenario 5 A patient presents with pH 7.4, pO2 12 kPa, pCO2 5.2 kPa, HCO3

- 22 mmol/litre. E - Chronic respiratory acidosis« YOUR ANSWER F - Normal acid/base balance« CORRECT ANSWER. The patient in this clinical scenario has a normal acid/base balance. 19. Theme: Risk of infection A Clean B Clean-contaminated C Contaminated D Dirty. From the list above, choose the most appropriate classification of each of the following procedures. Each answer may be used once, several times or not at all. Scenario 1 A 38-year-old male undergoing an elective repair of a right inguinal hernia. A - Clean« CORRECT ANSWER. An uncomplicated inguinal hernia repair should not involve any mucosal breach. This wound is deemed clean with a 2% risk of infection.

Page 18: Page | 1104

MyPasTest » MRCS A Online - Jan Exam 2015 14. Principles of Surgery in General; Perioperative Care (55Qs)

-------------------------------------------------------------------------------------------------------------------

---------------------------------------------------------------------------------------------------------------------------------------------------------------------------------------------------------------

Dr Mohammed Shamsul Islam Khan, Medical Officer, Clinical Neuro-Surgery, National Institute of Neuro-Sciences and Hospital

Sher-E-Bangla Nagar, Dhaka-1207, Bangladesh. Mobile: +880 1713 455 662, +880 1685 811979. E-mail: [email protected]

Page |

1121

Scenario 2 A 54-year-old male presents with sudden onset, severe upper abdominal pain and vomiting. An erect chest X-ray reveals a pneumoperitoneum. The working diagnosis is a perforated duodenal ulcer. He undergoes an emergency laparotomy. B - Clean-contaminated« YOUR ANSWER D - Dirty« CORRECT ANSWER. The erect chest X-ray demonstrates a pneumoperitoneum. This patient has chemical peritonitis from a perforated duodenal ulcer. There is extensive spillage resulting in a greater than 40% risk of infection. Scenario 3 A 15-year-old female presents with right iliac fossa pain, fever and vomiting. She has suspected acute appendicitis and is scheduled for an appendicectomy. C - Contaminated« YOUR ANSWER B - Clean-contaminated« CORRECT ANSWER Appendicectomy causes a mucosal breach. Providing there is no extensive spillage of gastrointestinal content, the risk of infection is 10%. Scenario 4 A 65-year-old female undergoes a sigmoid colectomy for diverticular disease. There is minimal spillage of bowel content. D - Dirty« YOUR ANSWER C - Contaminated« CORRECT ANSWER. A large bowel resection is likely to lead to some spillage of gastrointestinal content. In elective procedures this should be kept to a minimum wherever possible. In this instance the risk of infection ranges from 20% to 40%. Scenario 5 A 42-year-old female undergoing a laparoscopic cholecystectomy. E - « YOUR ANSWER B - Clean-contaminated« CORRECT ANSWER. Spillage from the biliary tract should be kept to a minimum as bile is extremely irritant. In an uncomplicated laparoscopic cholecystectomy the mucosa is breached and therefore the risk of infection is 10%. 20. Theme: Consent for surgery A Advance refusal B Battery C Best interests (treatment in) under common law D Consent for medical research E Implied consent (assent) F Informed consent G Negligence H Parental consent I Treatment under the Mental Health Act

Page 19: Page | 1104

MyPasTest » MRCS A Online - Jan Exam 2015 14. Principles of Surgery in General; Perioperative Care (55Qs)

-------------------------------------------------------------------------------------------------------------------

---------------------------------------------------------------------------------------------------------------------------------------------------------------------------------------------------------------

Dr Mohammed Shamsul Islam Khan, Medical Officer, Clinical Neuro-Surgery, National Institute of Neuro-Sciences and Hospital

Sher-E-Bangla Nagar, Dhaka-1207, Bangladesh. Mobile: +880 1713 455 662, +880 1685 811979. E-mail: [email protected]

Page |

1122

J Ward of court. The following descriptions all refer to issues concerning informed consent for surgery. Please select the most appropriate term from the above list to fit the scenario. The items may be used once, more than once, or not at all. Scenario 1 A 33-year-old glamour model with severe endometriosis underwent an abdominal hysterectomy/adhesiolysis for chronic pelvic pain. At the time of laparotomy, the uterus was found to be densely adherent to the rectum. The surgeon proceeded with the operation, resulting in a rectal perforation requiring repair and formation of a colostomy. Legal action is taken by the patient against the gynaecologist. A - Advance refusal « YOUR ANSWER B - Battery« CORRECT ANSWER. This lady has undergone a procedure without her consent and to which, had the procedure been abandoned, she would probably not have consented. Battery in principle is a violation of the civil law to touch another person without their consent. The harm resulting from battery does not necessarily have to be physical in nature. In the situation of informed consent harm may be construed as the moral violation of the patient’s right to exercise a rational choice. Scenario 2 A 63-year-old man underwent a laparoscopic cholecystectomy for symptomatic gallstone disease having previously had surgery for a perforated duodenal ulcer many years previously. Following a difficult procedure requiring laparoscopic adhesiolysis, the patient developed signs of an acute abdomen on the second post-operative day requiring laparotomy and small bowel resection for two perforations which had led to peritonitis. He had a stormy postoperative course on the Intensive Care Unit and eventually was left with a troublesome enterocutaneous fistula requiring further surgery. It is alleged subsequently that during consent, no mention was made of the risk of inadvertent injury to other structures such as bowel. B - Battery« YOUR ANSWER G - Negligence« CORRECT ANSWER. Battery is not the only legal action that surgeons risk for inadequately respecting their patient’s right to informed consent. Although the patient had given his general consent to the surgery in question, he had not been properly advised of its potential hazards (in this case inadvertent damage to other structures by trocar insertion/dissection/diathermy). In such situations, the legal claim is that had the patient known the risks in question, they would not have proceeded with the surgery. NB Surgeons are not protected from accusations of negligence by a signed consent form. Note: in this case, the surgeon might also be deemed negligent for choosing a laparoscopic approach to this surgery in the first place. Scenario 3 A 40-year-old women attends the colorectal outpatient clinic with rectal bleeding. Following history-taking and abdominal examination, the attending coloproctologist performed a rectal examination and proctosigmoidoscopy which the patient did not question, however the latter proved uncomfortable for the patient. The surgeon is surprised to find himself some time later responding to a complaint that the patient said she had not consented to the examination. C - Best interests (treatment in) under common law« YOUR ANSWER E - Implied consent (assent) « CORRECT ANSWER.

Page 20: Page | 1104

MyPasTest » MRCS A Online - Jan Exam 2015 14. Principles of Surgery in General; Perioperative Care (55Qs)

-------------------------------------------------------------------------------------------------------------------

---------------------------------------------------------------------------------------------------------------------------------------------------------------------------------------------------------------

Dr Mohammed Shamsul Islam Khan, Medical Officer, Clinical Neuro-Surgery, National Institute of Neuro-Sciences and Hospital

Sher-E-Bangla Nagar, Dhaka-1207, Bangladesh. Mobile: +880 1713 455 662, +880 1685 811979. E-mail: [email protected]

Page |

1123

In many situations it is neither necessary nor practicable to give informed (especially written) consent. It is deemed in such situations that the patient’s willingness to present themselves for investigation or treatment implies their consent (such as in this case). Scenario 4 A 23-year-old student suffering from a bipolar disorder attempts suicide by jumping in front of a train. He suffers bilateral pneumothoraces and a flail chest. On the ward, he attempts to remove his chest drains and essential oxygen therapy. He is currently under a section 2. D - Consent for medical research « YOUR ANSWER C - Best interests (treatment in) under common law« CORRECT ANSWER. In English law, no one is able to give consent for treatment of another adult against his or her will. The only exception to this rule is treatment of the mental disorder itself in patients sectioned under the 1983 Mental Health Act (answer I). This is not, however, true for treatment of other physical conditions in patients with mental illness. However, essential, especially life-saving, treatment may be given to such patients in the patients’ best interests under Common Law when they are deemed incapable for any reason to give informed consent. (Resources for this subject may be obtained from www.doh.gov.uk and the chapter by Len Doyal in the Clinical Surgery in General RCS Course manual.) 21. A 67-year-old man with hypertension requiring 2 antihypertensives is seen in the pre-assessment clinic. He gets out of breath when walking up two flights of stairs. What ASA grade is he?

ASA I« YOUR ANSWER

ASA II

ASA III« CORRECT ANSWER

ASA IV

ASA V. The American society of Anaesthesiologists (ASA) grade predicts a patients mortality based on medical co-morbidities. The correct answer is ASA III by virtue of the need for 2 anti-hypertensives he has severe systemic disease and his activity is also limited.

Definition Mortality %

I Normal healthy individual 0.06

II Mild systemic disease that does not limit activity 0.4

III Severe systemic disease that limits activity but is not incapacitating 4.5

IV Incapacitating systemic disease which is constantly life threatening 23

V Moribund, not expected to survive 24 hours with or without surgery 51

22. A 56-year-old man with a mental health disorder is seen in A&E with abdominal trauma and bleeding. He requires an urgent laparotomy but is deemed to be incapable of consent. Which of the following consent forms allows you to proceed to theatre?

A next of kin can give consent on his behalf« YOUR ANSWER

Page 21: Page | 1104

MyPasTest » MRCS A Online - Jan Exam 2015 14. Principles of Surgery in General; Perioperative Care (55Qs)

-------------------------------------------------------------------------------------------------------------------

---------------------------------------------------------------------------------------------------------------------------------------------------------------------------------------------------------------

Dr Mohammed Shamsul Islam Khan, Medical Officer, Clinical Neuro-Surgery, National Institute of Neuro-Sciences and Hospital

Sher-E-Bangla Nagar, Dhaka-1207, Bangladesh. Mobile: +880 1713 455 662, +880 1685 811979. E-mail: [email protected]

Page |

1124

An incapacity form can be issued by two doctors acting in his best interest« CORRECT ANSWER

As this is a life saving procedure consent is not required

Court approval should be sought prior to any intervention

He can be treated under the mental health act, issued by the duty psychiatrist. No one can either give or withhold consent of another adult. As this is an emergency life saving procedure, he can be taken to theatre with a consent 4 (England) or Incapacity form (Scotland) signed by his medical team. Although not strictly mandatory it is good practice to involve the next of kin if at all possible and practical. 23. Theme: Normal values for peripheral blood A 0.5-2.5% B 10 mm/h C 150-400 109 /l D 32-35 g/dl E 0.42-0.53 F 4.5-6.0 G 4-11 109 /l H 13-18 g/dl I 80-96 fl J 27-33 pg. For each of the options below, choose the single most likely diagnosis from the list above. Each option may be used once, more than once, or not at all. Scenario 1 Haemoglobin. A - 0.5-2.5%« YOUR ANSWER H - 13-18 g/dl« CORRECT ANSWER 13-18 g/dl. Scenario 2 Erythrocyte sedimentation rate. B - 10 mm/h« CORRECT ANSWER. Scenario 3 Packed cell volume. C - 150-400 109 /l« YOUR ANSWER E - 0.42-0.53« CORRECT ANSWER. Scenario 4 Platelet count. D - 32-35 g/dl« YOUR ANSWER C - 150-400 109 /l« CORRECT ANSWER. Scenario 5

Page 22: Page | 1104

MyPasTest » MRCS A Online - Jan Exam 2015 14. Principles of Surgery in General; Perioperative Care (55Qs)

-------------------------------------------------------------------------------------------------------------------

---------------------------------------------------------------------------------------------------------------------------------------------------------------------------------------------------------------

Dr Mohammed Shamsul Islam Khan, Medical Officer, Clinical Neuro-Surgery, National Institute of Neuro-Sciences and Hospital

Sher-E-Bangla Nagar, Dhaka-1207, Bangladesh. Mobile: +880 1713 455 662, +880 1685 811979. E-mail: [email protected]

Page |

1125

Mean corpuscular volume. E - 0.42-0.53« YOUR ANSWER I - 80-96 fl« CORRECT ANSWER. Scenario 6 Mean corpuscular haemoglobin concentration. F - 4.5-6.0« YOUR ANSWER D - 32-35 g/dl« CORRECT ANSWER. Scenario 7 White cell count. G - 4-11 109 /l« CORRECT ANSWER. Scenario 8 Mean corpuscular haemoglobin H - 13-18 g/dl« YOUR ANSWER J - 27-33 pg« CORRECT ANSWER. Scenario 9 Reticulocyte count I - 80-96 fl« YOUR ANSWER A - 0.5-2.5%« CORRECT ANSWER. 24. Theme: Perioperative care A Type I B Type II C Type III D Type IV E Type V. For each of the following patients, select the most likely type of hypersensitivity reaction. Each option may be used once, more than once, or not at all. Scenario 1 A 56-year-old lady with a diffuse goitre, proptosis, exompthalmos and pre-tibial myxoedema. A - Type I« YOUR ANSWER E - Type V« CORRECT ANSWER. Scenario 2 A 65-year-old patient presents with cough and increasing shortness of breath. Chest X-ray reveals a cavitating lesion in the left lung apex. Sputum is positive for acid-fast bacilli. B - Type II« YOUR ANSWER D - Type IV« CORRECT ANSWER. Scenario 3 A 25-year-old man who has peanut allergy unwittingly eats a piece of cake containing traces of nuts. He starts to have difficulty breathing, begins to wheeze and complains his throat is closing up.

Page 23: Page | 1104

MyPasTest » MRCS A Online - Jan Exam 2015 14. Principles of Surgery in General; Perioperative Care (55Qs)

-------------------------------------------------------------------------------------------------------------------

---------------------------------------------------------------------------------------------------------------------------------------------------------------------------------------------------------------

Dr Mohammed Shamsul Islam Khan, Medical Officer, Clinical Neuro-Surgery, National Institute of Neuro-Sciences and Hospital

Sher-E-Bangla Nagar, Dhaka-1207, Bangladesh. Mobile: +880 1713 455 662, +880 1685 811979. E-mail: [email protected]

Page |

1126

C - Type III« YOUR ANSWER A - Type I« CORRECT ANSWER. Scenario 4 A 25-year-old man complains of itchy, watery eyes and a runny nose. He is sensitive to pollen. D - Type IV« YOUR ANSWER A - Type I« CORRECT ANSWER. The Gell and Coomb’s classification (types I to V) is used to classify hypersensitivity reactions.• Type I reaction is mediated by immunoglobulin E (IgE) which binds to mast cells and basophils causing a release of inflammatory cytokines. The severity of the reaction can range from mild symptoms (e.g. a runny nose in hay fever) to anaphylactic shock (e.g. following a bee sting).• Type II reactions are mediated by antibodies against antigens on the cell surface. Examples include transfusion reactions and drug reactions.• Type III reactions are mediated by immune complex formation. Examples include systemic lupus erythematosus (SLE) and acute glomerulonephritis.• Type IV hypersensitively reactions are mediated by sensitised T lymphocytes. Tuberculosis is a good example.• Type V reactions involve antibodies produced with the property of stimulating specific cell targets (e.g. Grave's disease caused by antibodies that stimulate the thyroid-stimulating hormone receptor, leading to overactivity of the thyroid gland). 25. Theme: Thrombo-embolic prophylaxis A No specific treatment B Early mobilisation C Low-molecular weight heparin (LMWH) and thrombo-embolic disease stockings (TEDS) D High-dose LMWH and intra-operative pneumatic calf compression. E Systemic heparin therapy. From the list above, choose the most appropriate regimen to achieve appropriate thrombo-embolic prophylaxis for each of the following situations. Each answer may be used once, several times or not at all. Scenario 1 A 67-year-old male undergoing a sigmoid colectomy for malignancy. He has chronic obstructive pulmonary disease (COPD) and had a right total hip replacement 2 years previously. A - No specific treatment« YOUR ANSWER C - Low-molecular weight heparin (LMWH) and thrombo-embolic disease stockings (TEDS)« CORRECT ANSWER. This patient is undergoing a major procedure for malignancy. He has a moderate risk of thrombo-embolic disease (deep vein thrombosis (DVT) 10–40%, pulmonary embolism (PE) 0.1–1%). Scenario 2 A 26-year-old male undergoing a repair of a right inguinal hernia. B - Early mobilisation« CORRECT ANSWER. This patient is undergoing a minor procedure and providing it is uncomplicated it can be done as a day case procedure. Early mobilisation is sufficient unless he requires to stay as an inpatient.

Page 24: Page | 1104

MyPasTest » MRCS A Online - Jan Exam 2015 14. Principles of Surgery in General; Perioperative Care (55Qs)

-------------------------------------------------------------------------------------------------------------------

---------------------------------------------------------------------------------------------------------------------------------------------------------------------------------------------------------------

Dr Mohammed Shamsul Islam Khan, Medical Officer, Clinical Neuro-Surgery, National Institute of Neuro-Sciences and Hospital

Sher-E-Bangla Nagar, Dhaka-1207, Bangladesh. Mobile: +880 1713 455 662, +880 1685 811979. E-mail: [email protected]

Page |

1127

Scenario 3 A 64-year-old female who is undergoing a right hemicolectomy for a caecal tumour. She takes warfarin for a mechanical heart valve. C - Low-molecular weight heparin (LMWH) and thrombo-embolic disease stockings (TEDS)« YOUR ANSWER E - Systemic heparin therapy« CORRECT ANSWER. Any patient who has a mechanical heart valve should be given systemic heparin. This gives maximum protection to the valve. She can recommence warfarin when she is eating and drinking normally. Scenario 4 A 76-year-old male undergoing an abdominoperineal resection of rectum for a low lying rectal cancer. He had a previous deep vein thrombosis (DVT) following a femoral shaft fracture five years earlier D - High-dose LMWH and intra-operative pneumatic calf compression.« CORRECT ANSWER. This patient has had a previous DVT and has pelvic malignancy. He is at high risk of thrombo-embolic disease (DVT 40–80%, PE 1–10%). He should be given high-dose LMWH, and the use of intra-operative pneumatic calf compression device (eg Flowtrons) gives maximal prophylaxis. Scenario 5 A 37-year-old female who is undergoing a laparoscopic cholecystectomy. She has a body mass index (BMI) of 30. E - Systemic heparin therapy« YOUR ANSWER C - Low-molecular weight heparin (LMWH) and thrombo-embolic disease stockings (TEDS)« CORRECT ANSWER. This patient is at moderate risk of thrombo-embolic disease. She is obese and is undergoing an intermediate procedure. LMWH and TEDS are sufficient prophylaxis. 26. Theme: Pre-operative fitness for surgery. A Electrocardiogram only B Electrocardiogram and Urea & Electrolytes C Full blood count (FBC) only D FBC and Electrocardiogram E FBC, Electrocardiogram and Urea & Electrolytes F FBC, Electrocardiogram, Urea & Electrolytes and Chest X-ray G FBC, Electrocardiogram, Urea & Electrolytes, Chest X-ray and Lung Function Tests H FBC and Urea & Electrolytes I No investigation required J Urea & Electrolytes only. From the list above pick the single most appropriate answer containing the obligatory tests required for the following clinical scenarios. The items may be used once, more than once, or not at all. Scenario 1 A 37-year-old man who is ASA (American Society of Anaesthesiologists) Grade 1 is due to undergo a minor (Grade 1) procedure under general anaesthetic. A - Electrocardiogram only« YOUR ANSWER I - No investigation required« CORRECT ANSWER.

Page 25: Page | 1104

MyPasTest » MRCS A Online - Jan Exam 2015 14. Principles of Surgery in General; Perioperative Care (55Qs)

-------------------------------------------------------------------------------------------------------------------

---------------------------------------------------------------------------------------------------------------------------------------------------------------------------------------------------------------

Dr Mohammed Shamsul Islam Khan, Medical Officer, Clinical Neuro-Surgery, National Institute of Neuro-Sciences and Hospital

Sher-E-Bangla Nagar, Dhaka-1207, Bangladesh. Mobile: +880 1713 455 662, +880 1685 811979. E-mail: [email protected]

Page |

1128

Anyone between the ages of 16 and 39 years who is ASA (American Society of Anaesthesiologists) Grade 1 does not require any investigations prior to a minor procedure. The ASA grading system consists of five levels (1–5) and allows classification of a patient’s co-morbidity with respect to anaesthetic risk. Grade 1 is associated with minimum risk and includes healthy normal patients, ie with no clinically important co-morbidity and without a clinically significant past/present medical history. Scenario 2 A 48-year-old woman who is ASA Grade 2 secondary to cardiovascular disease requires a major (Grade 3) operation. B - Electrocardiogram and Urea & Electrolytes« YOUR ANSWER E - FBC, Electrocardiogram and Urea & Electrolytes« CORRECT ANSWER. These are the obligatory investigations for an ASA Grade 2 or Grade 3 patient with cardiovascular disease. Investigations that the NICE guidelines suggest should be considered include chest X-ray, urine analysis and blood gases. Scenario 3 An 83-year-old woman who is ASA Grade 2 secondary to renal disease requires a major, complex (Grade 4) procedure. C - Full blood count (FBC) only« YOUR ANSWER E - FBC, Electrocardiogram and Urea & Electrolytes« CORRECT ANSWER. FBC, electrocardiogram and urea & electrolytes are the minimum required for major, and major complex (Grade 3 and 4) types of surgery for most ASA grades 2 and 3. In this particular scenario, a chest X-ray, (as well as clotting studies, random glucose, blood gases and urine analysis) may also be considered. 27. Theme: Iatrogenic injuries A Femoral nerve B Genital branch of the genitofemoral nerve C Femoral branch of the genitofemoral nerve D Obturator nerve E Ilio-inguinal nerve. Injury to which of the above nerves would result in: (each option may be used once, more than once, or not at all) Scenario 1 Numbness of the scrotum and root of penis? A - Femoral nerve« YOUR ANSWER E - Ilio-inguinal nerve« CORRECT ANSWER. Scenario 2 Inability to extend the knee? B - Genital branch of the genitofemoral nerve« YOUR ANSWER A - Femoral nerve« CORRECT ANSWER. The femoral nerve (L2–4) passes through the substance of the psoas, and then under the inguinal ligament a finger’s breadth lateral to the femoral artery. Its branches are muscular to the anterior

Page 26: Page | 1104

MyPasTest » MRCS A Online - Jan Exam 2015 14. Principles of Surgery in General; Perioperative Care (55Qs)

-------------------------------------------------------------------------------------------------------------------

---------------------------------------------------------------------------------------------------------------------------------------------------------------------------------------------------------------

Dr Mohammed Shamsul Islam Khan, Medical Officer, Clinical Neuro-Surgery, National Institute of Neuro-Sciences and Hospital

Sher-E-Bangla Nagar, Dhaka-1207, Bangladesh. Mobile: +880 1713 455 662, +880 1685 811979. E-mail: [email protected]

Page |

1129

compartment of the thigh (quadriceps, sartorius, pectineus), cutaneous (skin of the medial side of leg, ankle, foot to the great toe through the saphenous nerve), and articular to the hip and knee joint. The ilio-inguinal nerve is the collateral branch of the ilio-hypogastric nerve, with a terminal distribution to the skin of the root of the penis and anterior third of the scrotum, and a small area below the medial end of the inguinal ligament. The genitofemoral nerve is derived from the first and second lumbar nerves but only fibres from L1 pass into the femoral branch. It supplies skin over most of the femoral triangle. The genital branch enters the spermatic cord and does not reach the skin. 28. Theme: Local anaesthetic agents A Amethocaine B Bupivacaine C Cinchocaine D Cocaine E Dibucaine F Prilocaine G None of above. For each scenario listed below, select the most appropriate local anaesthetic agent from the list of options above. Each option may be used once, more than once, or not at all. Scenario 1 This agent is commonly used for conjunctival anaesthesia A - Amethocaine « CORRECT ANSWER. Scenario 2 This agent has previously caused deaths when used in Bier’s blocks B - Bupivacaine « CORRECT ANSWER. Scenario 3 This agent causes sympathetic stimulation C - Cinchocaine « YOUR ANSWER D - Cocaine« CORRECT ANSWER. Amethocaine is an ester that rapidly diffuses into the conjunctiva. Cocaine is also an ester but causes sympathetic stimulation, and so is reserved for situations where vasoconstriction is required (eg nasal procedures). All the other local anaesthetics are amides. Bupivacaine binds to the myocardium and has caused a number of deaths when used in Bier’s blocks. Thus, it is now contraindicated for use in Bier’s blocks. Prilocaine binds poorly to the myocardium and so is the best agent for intravenous regional anaesthesia. 29. Theme: Timing of surgical procedures A Urgent operation within next 12–24 hours B Wait 2 weeks and operate C Wait 6 months and operate D Wait 6 weeks and operate. For each of the following scenarios, select the most likely answer from the above list. Each option may be used once, more than once, or not at all.

Page 27: Page | 1104

MyPasTest » MRCS A Online - Jan Exam 2015 14. Principles of Surgery in General; Perioperative Care (55Qs)

-------------------------------------------------------------------------------------------------------------------

---------------------------------------------------------------------------------------------------------------------------------------------------------------------------------------------------------------

Dr Mohammed Shamsul Islam Khan, Medical Officer, Clinical Neuro-Surgery, National Institute of Neuro-Sciences and Hospital

Sher-E-Bangla Nagar, Dhaka-1207, Bangladesh. Mobile: +880 1713 455 662, +880 1685 811979. E-mail: [email protected]

Page |

1130

Scenario 1 A 45-year-old lady is brought to the Emergency Department with an 18-hour history of right upper quadrant pain, and a tender mass in this region. Temperature is 39 ºC and ultrasound demonstrates a distended gallbladder with pericholecystic fluid. The WBC count is 26 x 109/l. The patient remains pyrexial and tachycardic after 24 hours of antibiotics. A - Urgent operation within next 12–24 hours« CORRECT ANSWER. This lady has severe acute cholecystitis, which is not responding to antibiotics. The concern here is a possible empyema or gangrenous cholecystitis. Following urgent resuscitation, an emergency cholecystectomy should be performed. This may be done laparoscopically or by open operation depending on the experience and skill of the operating surgeon. ‘Hot’ gallbladders, ie within the first 72 hours can be relatively easy to remove as the tissue oedema aids the dissection. However, if there have been recurrent episodes of inflammation or if the initial episode has been longer than 5 days, then surgery may be very difficult and a delay in operation should be considered. Scenario 2 A 12-year-old boy is brought to the Emergency Department with a 3-day history of right iliac fossa pain. There is a palpable mass with only localised tenderness. Temperature and pain respond to intravenous antibiotics. B - Wait 2 weeks and operate« YOUR ANSWER D - Wait 6 weeks and operate« CORRECT ANSWER. An appendix abscess can be treated conservatively with intravenous antibiotics as long as the patient responds to treatment. Indications for surgery are a persistent pyrexia, tachycardia and generalised peritonitis. The optimal time for an interval appendicectomy is debatable, but 6 weeks is a reasonable time period. Scenario 3 A 34-year-old lady with known Crohn’s disease of the terminal ileum undergoes a right hemicolectomy. Unfortunately, 9 days post-operation she develops two enterocutaneous fistulas. She is otherwise well. C - Wait 6 months and operate« CORRECT ANSWER. The management of a post-operative enterocutaneous fistula is based initially on control of sepsis, nutrition (enteral or parenteral) and wound management. Subsequent anatomical mapping of the fistula can be performed at a later stage when revision surgery is planned. After the first post-operative week, re-entering the abdomen is difficult and the patient may not have a peritoneal cavity. This patient appears to have a controlled fistula. It is therefore prudent to refrain from immediate surgery and manage her according to the above principles. Re-operating before a minimum of 3 months is fraught with risk of intestinal injury; therefore a period of 6 months is the best option here. Scenario 4 A 79-year-old man is admitted with a sigmoid perforation secondary to diverticular disease. He had a Hartmann’s procedure and makes a satisfactory post-operative recovery. He is extremely keen for a reversal and asks when this procedure might be feasible. D - Wait 6 weeks and operate« YOUR ANSWER C - Wait 6 months and operate« CORRECT ANSWER.

Page 28: Page | 1104

MyPasTest » MRCS A Online - Jan Exam 2015 14. Principles of Surgery in General; Perioperative Care (55Qs)

-------------------------------------------------------------------------------------------------------------------

---------------------------------------------------------------------------------------------------------------------------------------------------------------------------------------------------------------

Dr Mohammed Shamsul Islam Khan, Medical Officer, Clinical Neuro-Surgery, National Institute of Neuro-Sciences and Hospital

Sher-E-Bangla Nagar, Dhaka-1207, Bangladesh. Mobile: +880 1713 455 662, +880 1685 811979. E-mail: [email protected]

Page |

1131

Reversal of a Hartmann’s procedure may be more difficult than the initial bowel resection because of the presence of dense adhesions and identification of the rectal stump. Therefore, it is suggested to wait for a period of 6 months before attempting a reversal as, by that time, the adhesions may be more conducive to surgery. 30. A 61-year-old asthmatic is sent for peak flow rates as part of his pre-assessment. What result would you expect?

FVC is reduced but FEV1/FVC is normal« YOUR ANSWER

FVC normal or reduced and FEV1/FVC is reduced « CORRECT ANSWER

Normal FVC and normal FEV1/FVC

All of the above

None of the above. In asthma (an obstructive lung disorder) the forced expiratory volume in 1 second (FEV1) is usually decreased, the forced vital capacity (FVC) is usually normal and the ratio FEV1/FVC is decreased. 31. Theme: Blood transfusion complications A Air embolism B Allergic reaction C Bacterial contamination D Cannulation site thrombophlebitis E Circulatory overload F Delayed haemolytic transfusion reaction G Graft-versus-host disease H Hyperkalaemia I Hypocalcaemia J Hypothermia K Immediate haemolytic transfusion reaction L Immunosuppression M Iron overload N Non-haemolytic febrile transfusion reaction O Post-transfusion purpura P Transfusion-related acute lung injury (TRALI) Q Transfusion-transmitted infection R Urticaria. The following patients all have complications of blood transfusion. Please select the most appropriate cause or description from the above list. The items may be used once, more than once, or not at all. Scenario 1 You are called to see a 35-year-old multiparous woman on your surgical ward who, 6 days after an intra-operative blood transfusion, is complaining of nose bleeds and sacral bruising. A coagulation screen is normal. Her platelet count is 28 x 109/litre. A - Air embolism « YOUR ANSWER O - Post-transfusion purpura« CORRECT ANSWER.

Page 29: Page | 1104

MyPasTest » MRCS A Online - Jan Exam 2015 14. Principles of Surgery in General; Perioperative Care (55Qs)

-------------------------------------------------------------------------------------------------------------------

---------------------------------------------------------------------------------------------------------------------------------------------------------------------------------------------------------------

Dr Mohammed Shamsul Islam Khan, Medical Officer, Clinical Neuro-Surgery, National Institute of Neuro-Sciences and Hospital

Sher-E-Bangla Nagar, Dhaka-1207, Bangladesh. Mobile: +880 1713 455 662, +880 1685 811979. E-mail: [email protected]

Page |

1132

PTP is defined as thrombocytopenia arising 5–12 days after transfusion of red cells. It is associated with the presence in the recipient of antibodies directed against the human platelet antigen (HPA) system. The patient becomes sensitised to foreign platelet antigen (commonly anti-PlA1), usually as a result of previous pregnancy or, more rarely, transfusion. If the patient then receives subsequent blood carrying this antigen, a secondary response occurs, leading to destruction of the recipient’s own platelets. Clinical signs are purpura and mucosal bleeding. Optimal treatment is by means of large doses of intravenous immunoglobulin, or plasma exchange. Scenario 2 A 40-year-old man, with no history of pulmonary disease, develops acute dyspnoea within 16 h of receiving a whole blood transfusion. His oxygen saturations on air are 90%. Arterial blood gas pA(O2) is 9.0 kPa. A chest X-ray shows patchy shadowing in both lung fields. B - Allergic reaction« YOUR ANSWER P - Transfusion-related acute lung injury (TRALI)« CORRECT ANSWER. This man is suffering from TRALI. This is defined as acute dyspnoea with hypoxia and bilateral pulmonary infiltrates occurring within 24 h of transfusion. When there is no other apparent cause (too delayed to be air embolus) such symptoms point to this as the most likely diagnosis. TRALI is caused by agglutination of the patient’s white blood cells when they are exposed to potent HLA antibodies in the blood donor’s plasma. Diagnosis is supported by a positive cytotoxic cross-match where the donor’s plasma reacts to the patient’s white cells. Management consists of supportive measures, such as stopping the transfusion (if the reaction occurs while this is ongoing), resuscitation, oxygen therapy and Intensive Care Unit intervention, with or without ventilation as necessary. Scenario 3 A 24-year-old female pedestrian is admitted unconscious to the Intensive Care Unit after a hit-and-run accident. She has been appropriately assessed and managed for her injuries. A blood transfusion has just been commenced. Having remained previously stable, you now notice that she is spiking a temperature of 39.5 °C, her blood pressure is 100/60 mmHg, and she is bleeding profusely through her wound dressings and oozing blood around her venepuncture sites. Her catheter contains dark urine that is dipstick positive for haemoglobinuria. C - Bacterial contamination « YOUR ANSWER K - Immediate haemolytic transfusion reaction« CORRECT ANSWER. Clinical symptoms of a major transfusion reaction include urticaria, abdominal/lumbar/chest pain, flushing, headache, dyspnoea, vomiting and rigors. Clearly these signs are far easier to elicit in the conscious patient. In this scenario, however, the woman is not able to describe such symptoms. The signs in such a patient are therefore of utmost importance in diagnosing a haemolytic reaction. In the unconscious patient, hypotension (a fall > 10 mmHg) and generalised uncontrollable bleeding are the most important signs. Alongside these, further indications of an immediate haemolytic reaction are fever (an increase of more than 1.5 °C above baseline), jaundice and haemoglobinuria. Immediate haemolytic transfusion reaction is usually the result of ABO incompatibility, leading to complement activation and massive intravascular haemolysis. Most severe reactions occur because of clerical error, hence all documentation should be re-checked and both patient and donor blood (from the bag in use) should undergo repeat cross-matching. The prime objective in the acute management of such a reaction is to maintain blood pressure and renal perfusion. This is achieved by immediate cessation of

Page 30: Page | 1104

MyPasTest » MRCS A Online - Jan Exam 2015 14. Principles of Surgery in General; Perioperative Care (55Qs)

-------------------------------------------------------------------------------------------------------------------

---------------------------------------------------------------------------------------------------------------------------------------------------------------------------------------------------------------

Dr Mohammed Shamsul Islam Khan, Medical Officer, Clinical Neuro-Surgery, National Institute of Neuro-Sciences and Hospital

Sher-E-Bangla Nagar, Dhaka-1207, Bangladesh. Mobile: +880 1713 455 662, +880 1685 811979. E-mail: [email protected]

Page |

1133

the transfusion and supportive resuscitation with intravenous fluids, titration of (<st1:time ">1 : 10, 000 adrenaline as well as chlorpheniramine (10 mg) and hydrocortisone (100 mg). Renal failure and disseminated intravascular coagulation may lead to death in the worst case scenario. Scenario 4 A 26-year-old Afro-Caribbean woman presents to her general practitioner (GP) with symptoms of lethargy and weight gain. She is known to have sickle cell anaemia and, because of multiple painful crises, has required regular blood transfusions. Blood tests requested by her GP show: thyroid-stimulating hormone 9 µmol/litre, free thyroxine 7 pmol/litre, calcium 2.15 mmol/litre. Further investigations reveal poor cardiac function. D - Cannulation site thrombophlebitis « YOUR ANSWER M - Iron overload« CORRECT ANSWER. Repeated red cell transfusions (often given frequently as prophylaxis to sickle-cell sufferers having regular crises, to suppress haemoglobin S production) can lead to iron overload in the absence of chelation. Each unit of blood contains between 200 and 250 mg of iron. After 12 g of iron have been transfused (approximately 50 units of blood), clinical abnormalities start to appear. Unfortunately, organ damage will have occurred earlier than this. Iron damages the liver, the endocrine organs (leading to failure of growth, delayed or absent puberty, diabetes mellitus, hypothyroidism and hypoparathyroidism) and the myocardium. Parenteral treatment with desferrioxamine can prevent iron overload, if compliance with medication is good. In the absence of intensive iron chelation, death often occurs in the second or third decade, usually from congestive heart failure or cardiac arrhythmias. 32. A 56-year-old man is admitted to A&E with a compound tibial fracture. You are asked to book him for theatre urgently, in order to preserve the limb. Which CEPOD classification do you use?

CEPOD 1« YOUR ANSWER

CEPOD 2« CORRECT ANSWER

CEPOD 3

CEPOD 4

CEPOD 5. The NCEPOD classification of intervention rationalizes available theatre resources and delivers them to those who require care by urgency of injuries. Compound fractures should be washed out within 6 hours and thus fall into category 2. CEPOD (confidential enquiry into patient outcome and death) scores are:

01. IMMEDIATE – Immediate life, limb or organ-saving intervention – resuscitation simultaneous with intervention. Normally within minutes of decision to operate.

a. Life-saving b. Other e.g. limb or organ saving

02. URGENT – Intervention for acute onset or clinical deterioration of potentially life-threatening conditions, for those conditions that may threaten the survival of limb or organ, for fixation of many fractures and for relief of pain or other distressing symptoms. Normally within hours of decision to operate.

03. EXPEDITED – Patient requiring early treatment where the condition is not an immediate threat to life, limb or organ survival. Normally within days of decision to operate.

Page 31: Page | 1104

MyPasTest » MRCS A Online - Jan Exam 2015 14. Principles of Surgery in General; Perioperative Care (55Qs)

-------------------------------------------------------------------------------------------------------------------

---------------------------------------------------------------------------------------------------------------------------------------------------------------------------------------------------------------

Dr Mohammed Shamsul Islam Khan, Medical Officer, Clinical Neuro-Surgery, National Institute of Neuro-Sciences and Hospital

Sher-E-Bangla Nagar, Dhaka-1207, Bangladesh. Mobile: +880 1713 455 662, +880 1685 811979. E-mail: [email protected]

Page |

1134

04. ELECTIVE – Intervention planned or booked in advance of routine admission to hospital. Timing to suit patient, hospital and staff

05. Does not exist!

As described above, an open compound tibial fracture would be considered as needing URGENT limb-saving surgery (i.e. CEPOD 2). Please see the following link for further details: http://www.ncepod.org.uk/pdf/NCEPODClassification.pdf 33. Theme: Acid–base balance A Metabolic acidosis B Metabolic alkalosis C Normal pH D Respiratory acidosis E Respiratory alkalosis. For each of the patients described below, select the most likely acid–base status from the list of options above. Each option may be used once, more than once, or not at all. Scenario 1 A 70-year-old man with chronic obstructive pulmonary disease collapses on the ward after being admitted with an exacerbation of infection. A - Metabolic acidosis « YOUR ANSWER D - Respiratory acidosis« CORRECT ANSWER. Patients who suffer from chronic obstructive pulmonary disease (COPD) usually have a respiratory acidosis with a compensatory metabolic alkalosis. In cases where there is an infective exacerbation, respiratory acidosis becomes the predominant feature with marked CO2 retention. Scenario 2 A 65-year-old man admitted with epigastric pain, vomiting and jaundice. A serum amylase level of 2000 IU was recorded. B - Metabolic alkalosis « YOUR ANSWER A - Metabolic acidosis « CORRECT ANSWER. A serum amylase level of four times the normal value is pathognomic of pancreatitis. Patients with severe acute pancreatitis will usually be acidotic from a number of causes: hypovolaemic shock, acute renal failure and lactic acidosis. Scenario 3 A 41-year-old woman with shortness of breath and haemoptysis, and complaining of chest pain, was noted to have unilateral leg swelling 4–6 months after a right upper lobectomy for carcinoma of the lung. C - Normal pH« YOUR ANSWER E - Respiratory alkalosis« CORRECT ANSWER. The third case is indicative of a pulmonary embolism (PE). The first indication of a PE is dyspnoea. The patient will be hyperventilating, so producing a respiratory alkalosis. The issue of ‘lobectomy’ in this scenario should not confuse the candidate, since the lobectomy had been performed months previously,

Page 32: Page | 1104

MyPasTest » MRCS A Online - Jan Exam 2015 14. Principles of Surgery in General; Perioperative Care (55Qs)

-------------------------------------------------------------------------------------------------------------------

---------------------------------------------------------------------------------------------------------------------------------------------------------------------------------------------------------------

Dr Mohammed Shamsul Islam Khan, Medical Officer, Clinical Neuro-Surgery, National Institute of Neuro-Sciences and Hospital

Sher-E-Bangla Nagar, Dhaka-1207, Bangladesh. Mobile: +880 1713 455 662, +880 1685 811979. E-mail: [email protected]

Page |

1135

allowing the residual lung tissue to have re-expanded adequately to overcome any initial V-Q deficits. The acute problem therefore is the PE, which, due to hyperventilation, is likely to incur a respiratory alkalosis. 34. Theme: Haematology A Capillary abnormality B Abnormal prothrombin C Vitamin K deficiency D Increased fibrinolysis E Factor VIII:C deficiency. For each of the scenarios match the most appropriate answer. Each option may be used once, more than once, or not at all. Scenario 1 A 60-year-old patient on warfarin for aortic valve replacement presents with haematuria and an international normalised ratio (INR) of 4.2. A - Capillary abnormality« YOUR ANSWER C - Vitamin K deficiency« CORRECT ANSWER. Scenario 2 A young child with no past medical history had bleeding at a tooth socket after tooth extractions. This stops initially but later re-occurs. B - Abnormal prothrombin« YOUR ANSWER E - Factor VIII:C deficiency« CORRECT ANSWER. Warfarin has a similar structure to vitamin K and acts as a competitive inhibitor of epoxide reductase, thereby reducing the availability of vitamin K to form factors II, VII, IX and X. Warfarin is monitored by measurement of the prothrombin time (INR). Factor VIII:C deficiency (von Willebrand’s disease) is an absence or low level of plasma VIII clotting activity. Prolonged bleeding after dental extractions is common. 35. Theme: Perioperative care A The left recurrent laryngeal nerve B The left superior laryngeal nerve C The left chorda tympani branch of the facial nerve D The left mandibular branch of the facial nerve E The left cervical branch of the facial nerve. Which one of the above nerves has been most likely affected in the following patients. Each option may be used once, more than once, or not at all. Scenario 1 A 45-year-old woman who underwent a left stapedectomy for otosclerosis. Post-operatively, she complains of alteration in taste. Gross examination of the facial nerve seems fine A - The left recurrent laryngeal nerve« YOUR ANSWER C - The left chorda tympani branch of the facial nerve« CORRECT ANSWER.

Page 33: Page | 1104

MyPasTest » MRCS A Online - Jan Exam 2015 14. Principles of Surgery in General; Perioperative Care (55Qs)

-------------------------------------------------------------------------------------------------------------------

---------------------------------------------------------------------------------------------------------------------------------------------------------------------------------------------------------------

Dr Mohammed Shamsul Islam Khan, Medical Officer, Clinical Neuro-Surgery, National Institute of Neuro-Sciences and Hospital

Sher-E-Bangla Nagar, Dhaka-1207, Bangladesh. Mobile: +880 1713 455 662, +880 1685 811979. E-mail: [email protected]

Page |

1136

Scenario 2 A 47-year-old opera singer underwent a left hemithyroidectomy for a follicular lesion identified on FNA. Postoperatively she complains that her voice seems different. Flexible laryngoscopy shows a paralysed left vocal cord. B - The left superior laryngeal nerve« YOUR ANSWER A - The left recurrent laryngeal nerve« CORRECT ANSWER. Scenario 3 A 47-year-old opera singer underwent a left hemithyroidectomy for a follicular lesion identified on FNA. Postoperatively she complains her voice seems different. Flexible laryngoscopy shows the left vocal cord appears to be moving normally. C - The left chorda tympani branch of the facial nerve« YOUR ANSWER B - The left superior laryngeal nerve« CORRECT ANSWER. Scenario 4 A 56-year-old lady had a left submandibular gland excision following suspicion of malignancy on previous FNA of the gland. Postoperatively, she has difficulty sipping water. The left lower lip appears to be drooping. D - The left mandibular branch of the facial nerve« CORRECT ANSWER. The recurrent laryngeal nerve is a branch of the vagus nerve. It supplies all the intrinsic muscles of the larynx except the cricothyroid. Injury to it can completely paralyse the vocal cord. The cricothyroid muscle is supplied by the superior laryngeal nerve. Injury to it does not completely paralyse the vocal cord. It affects high-frequency speech and therefore can often go unnoticed. These nerves are at particular risk during thyroid surgery.The facial nerve divides into five terminal branches: temporal, zygomatic, buccal, marginal and cervical. The marginal branch is particularly at risk during submandibular gland surgery. 36. You are in the pre-assessment clinic and see a 56-year-old lady who is due to undergo an elective cholecystectomy. Of the following, which is an indicator for doing urea and electrolyte blood tests?

Age over 50« YOUR ANSWER

All women

Diabetics« CORRECT ANSWER

Respiratory disorders

Those undergoing prolonged anaesthetic time > 60 minutes. Patients over 60 years, those with cardiovascular and renal disease, diabetics and those on steroids/ACEI or diuretics should have U+E’s monitored pre-operatively. 37. Theme: Principles of surgery - perioperative care A Erect chest X-ray B Plain abdominal X-ray C Abdominal and pelvic ultrasound scan D Contrast computerised tomography (CT) scan of the abdomen and pelvis E Small Bowel meal F Contrast enema

Page 34: Page | 1104

MyPasTest » MRCS A Online - Jan Exam 2015 14. Principles of Surgery in General; Perioperative Care (55Qs)

-------------------------------------------------------------------------------------------------------------------

---------------------------------------------------------------------------------------------------------------------------------------------------------------------------------------------------------------

Dr Mohammed Shamsul Islam Khan, Medical Officer, Clinical Neuro-Surgery, National Institute of Neuro-Sciences and Hospital

Sher-E-Bangla Nagar, Dhaka-1207, Bangladesh. Mobile: +880 1713 455 662, +880 1685 811979. E-mail: [email protected]

Page |

1137

G Magnetic resonance imaging (MRI) of the abdomen and pelvis. From the list above, choose the most appropriate radiological investigation for the following scenarios. Each answer may be used once, more than once, or not at all. Scenario 1 A 25-year-old female presents with a 2-day history of right iliac fossa pain which is sharp in nature. She has a low-grade pyrexia with a normal white cell count and C-reactive protein (CRP). She is midway through her menstrual cycle. A - Erect chest X-ray« YOUR ANSWER C - Abdominal and pelvic ultrasound scan« CORRECT ANSWER. In this case an ultrasound scan will aid the diagnosis. While appendicitis remains a possibility, there are several other potential causes of right iliac fossa pain. In women of child-bearing years, plain X-rays should be avoided where possible. Scenario 2 A 42-year-old man known to have Crohn's disease for the last 12 years presents to the outpatients clinic. He describes episodic lower abdominal pain which is colicky in nature. His appetite is reduced and he has lost approximately 7 kg in weight. B - Plain abdominal X-ray« YOUR ANSWER E - Small Bowel meal« CORRECT ANSWER. This patient is known to have Crohn's disease and from his history of abdominal pain and weight loss he is clearly symptomatic. Often the small bowel is involved and can develop strictures. This is best seen by performing a contrast (barium) meal. Scenario 3 A 58-year-old female presents as an emergency with central and lower abdominal pain and vomiting for the last 12h. On examination her abdomen is distended and tympanic with active bowel sounds. She has previously had an appendicectomy in her childhood. C - Abdominal and pelvic ultrasound scan« YOUR ANSWER B - Plain abdominal X-ray« CORRECT ANSWER. This patient is presenting with symptoms and signs of an intestinal obstruction. Given the rapid onset of symtoms there is probably obstruction of the small bowel. Given the history of previous appendicectomy, the cause may well be adhesions. A plain abdominal X-ray will demonstrate the features of bowel obstruction. Scenario 4 A 74-year-old man presents as an emergency with lower abdominal pain and diarrhoea for the last two days. He describes altered blood mixed with his stool. On examination there is a firm and tender mass in the left iliac fossa. He has a pyrexia of 38.2°C with an elevated white cell count and CRP. D - Contrast computerised tomography (CT) scan of the abdomen and pelvis« CORRECT ANSWER. In this case where there is a palpable mass in the left iliac fossa which is tender, along with a pyrexia and an elevated white cell count and C-reactive protein (CRP), the likely diagnosis is that of acute diverticulitis. This can form an abscess. This is optimally visualised by contrast CT scan.

Page 35: Page | 1104

MyPasTest » MRCS A Online - Jan Exam 2015 14. Principles of Surgery in General; Perioperative Care (55Qs)

-------------------------------------------------------------------------------------------------------------------

---------------------------------------------------------------------------------------------------------------------------------------------------------------------------------------------------------------

Dr Mohammed Shamsul Islam Khan, Medical Officer, Clinical Neuro-Surgery, National Institute of Neuro-Sciences and Hospital

Sher-E-Bangla Nagar, Dhaka-1207, Bangladesh. Mobile: +880 1713 455 662, +880 1685 811979. E-mail: [email protected]

Page |

1138

Scenario 5 A 55-year-old man presents with 8h of upper abdominal pain. The onset was sudden and has been steadily worsening. He has a tachycardia and a raised white cell count. He takes diclofenac for chronic back pain. E - Small Bowel meal« YOUR ANSWER A - Erect chest X-ray« CORRECT ANSWER. These symptoms should lead to immediate suspicion of a perforation. The patient also takes a non-steroidal anti-inflammatory drug (NSAID) which is known to predispose to peptic ulceration. In this case where a perforated duodenal ulcer is likely, an erect chest X-ray will demonstrate a pneumoperitoneum in approximately 60-80% of cases. 38. A 21-year-old insulin dependent diabetic is seen in clinic. He is due to undergo elective circumcision under general anaesthetic. Which of the following measures need to be considered?

He should be placed first on the theatre list« YOUR ANSWER

He should have a GKI infusion, this should be taken down in recovery

He should have addition oral hypoglycaemics on the morning of his surgery

He should not be fasted for 6 hours prior to surgery

He should remain on long acting insulin. Diabetics should be placed first on the list to prevent prolonged starvation. GKI infusions should remain up until the patient is eating and drinking normally. One overnight inpatient stay must be anticipated. The requirement for fasting is dependent on the general anaesthetic rather than comorbidities. If anything this is more important in diabetics as they often have a degree of gastroparesis. 39. Theme: Acid-base balance A Metabolic acidosis B Metabolic alkalosis C Normal pH D Respiratory acidosis E Respiratory alkalosis. For each of the scenarios given choose match the most appropriate answer. Each option may be used once, more than once, or not at all. Scenario 1 A 70-year-old man with chronic obstructive airway disease who collapsed on the ward after being admitted with an infective exacerbation. A - Metabolic acidosis« YOUR ANSWER D - Respiratory acidosis« CORRECT ANSWER. Patients who suffer from chronic obstructive pulmonary disease usually have a respiratory acidosis with compensatory metabolic alkalosis. In cases where there is an infective exacerbation, respiratory acidosis becomes the predominant feature with marked CO2 retention. Scenario 2 A 65-year-old man admitted with epigastric pain and jaundice; serum amylase of 2000 IU recorded.

Page 36: Page | 1104

MyPasTest » MRCS A Online - Jan Exam 2015 14. Principles of Surgery in General; Perioperative Care (55Qs)

-------------------------------------------------------------------------------------------------------------------

---------------------------------------------------------------------------------------------------------------------------------------------------------------------------------------------------------------

Dr Mohammed Shamsul Islam Khan, Medical Officer, Clinical Neuro-Surgery, National Institute of Neuro-Sciences and Hospital

Sher-E-Bangla Nagar, Dhaka-1207, Bangladesh. Mobile: +880 1713 455 662, +880 1685 811979. E-mail: [email protected]

Page |

1139

B - Metabolic alkalosis« YOUR ANSWER A - Metabolic acidosis« CORRECT ANSWER. A serum amylase of four times the normal value is pathognomonic of pancreatitis. Patients with severe acute pancreatitis will usually be acidotic from a number of causes: hypovolaemic shock, acute renal failure and lactic acidosis. Scenario 3 A 41-year-old woman with shortness of breath and haemoptysis, who complained of chest pain, was noted to have unilateral leg swelling 4–6 months after a right upper lobectomy for carcinoma of lung. C - Normal pH« YOUR ANSWER E - Respiratory alkalosis« CORRECT ANSWER. This case is representative of a pulmonary embolism (PE). The first indication of a PE is dyspnoea. The patient will be hyperventilating and thus produce a respiratory alkalosis. Following a lobectomy. 40. Theme: Diabetes in surgery A Continue oral antiglycaemic(s) B Omit morning dose of oral antiglycaemic(s) C Continue subcutaneous insulin D Omit morning dose of subcutaneous insulin E Commence glucose potassium insulin (GKI) sliding scale regimen F No specific therapy. From the list above, choose the most appropriate regimen for a diabetic patient on the morning of surgery. Each answer may be used once, several times or not at all. Scenario 1 A patient who uses the oral antiglycaemics gliclazide and metformin undergoing an elective inguinal hernia repair. B - Omit morning dose of oral antiglycaemic(s)« CORRECT ANSWER This procedure is relatively minor. Providing it is uncomplicated the patient can recommence oral fluids and diet as tolerated on return from theatre. Scenario 2 A patient who has insulin-dependent diabetes and is undergoing a small bowel resection for a Crohn’s stricture. A - Continue oral antiglycaemic(s)« YOUR ANSWER E - Commence glucose potassium insulin (GKI) sliding scale regimen« CORRECT ANSWER. This patient is undergoing a major procedure that is likely to lead to a period of ileus. The GKI regimen should be continued until the patient is eating and drinking normally. Scenario 3 A patient who has diet-controlled diabetes who is undergoing a repair of a small paraumbilical hernia. C - Continue subcutaneous insulin« YOUR ANSWER F - No specific therapy« CORRECT ANSWER. This patient has diet-controlled diabetes and is undergoing a minor procedure, ie short duration. No specific therapy is indicated.

Page 37: Page | 1104

MyPasTest » MRCS A Online - Jan Exam 2015 14. Principles of Surgery in General; Perioperative Care (55Qs)

-------------------------------------------------------------------------------------------------------------------

---------------------------------------------------------------------------------------------------------------------------------------------------------------------------------------------------------------

Dr Mohammed Shamsul Islam Khan, Medical Officer, Clinical Neuro-Surgery, National Institute of Neuro-Sciences and Hospital

Sher-E-Bangla Nagar, Dhaka-1207, Bangladesh. Mobile: +880 1713 455 662, +880 1685 811979. E-mail: [email protected]

Page |

1140

Scenario 4 A patient with insulin-dependent diabetes who is undergoing a procedure for removal of varicose veins. D - Omit morning dose of subcutaneous insulin« CORRECT ANSWER. When minor procedures are performed on insulin-dependent patients, a glucose potassium insulin (GKI) regimen is not always required. If the procedure is likely to be of short duration and the patient is placed first on the operating list, omission of the morning dose of subcutaneous insulin should be sufficient. Normal diet can recommence on return from theatre. Scenario 5 A patient who takes metformin and is undergoing a laparoscopic cholecystectomy. E - Commence glucose potassium insulin (GKI) sliding scale regimen« YOUR ANSWER B - Omit morning dose of oral antiglycaemic(s)« CORRECT ANSWER. Omission of the morning dose is usually sufficient in these patients undergoing short procedures. Normal diet and medications can be continued on return from theatre. 41. Theme: Perioperative care A ASA I B ASA II C ASA III D ASA IV E ASA V. Chose the most appropriate American Society of Anaesthesiologists (ASA) grade for the following patients. Each option may be used once, more than once, or not at all. Scenario 1 A young, healthy marathon runner who is admitted for elective tonsillectomy for recurrent tonsillitis A - ASA I« CORRECT ANSWER. Scenario 2 A 60-year-old gentleman who is admitted for elective inguinal hernia repair. He uses his glyceryl trinitrate (GTN) spray once a month prior to mowing the lawn. B - ASA II« CORRECT ANSWER. Scenario 3 An 80-year-old lady admitted through the emergency department with a fractured neck of femur. She has a history of emphysema, is normally bed-bound and desaturates to 80% oxygen saturation without her home oxygen therapy. C - ASA III« YOUR ANSWER D - ASA IV« CORRECT ANSWER. Scenario 4 A 70-year-old gentleman admitted acutely with general peritonitis. Clinically, he is in septic shock, has features of disseminated intravascular coagulation (DIC) and is developing multi-organ failure. He is not expected to survive overnight.

Page 38: Page | 1104

MyPasTest » MRCS A Online - Jan Exam 2015 14. Principles of Surgery in General; Perioperative Care (55Qs)

-------------------------------------------------------------------------------------------------------------------

---------------------------------------------------------------------------------------------------------------------------------------------------------------------------------------------------------------

Dr Mohammed Shamsul Islam Khan, Medical Officer, Clinical Neuro-Surgery, National Institute of Neuro-Sciences and Hospital

Sher-E-Bangla Nagar, Dhaka-1207, Bangladesh. Mobile: +880 1713 455 662, +880 1685 811979. E-mail: [email protected]

Page |

1141

D - ASA IV« YOUR ANSWER E - ASA V« CORRECT ANSWER. The table below gives a summary of each of the American Society of Anaesthesiologists (ASA) grades.

ASA grade Definition Mortality (%)

I Normal healthy individual 0.05

II Mild systemic disease that does not limit activity 0.4

III Severe systemic disease that limits activity but is not incapacitating 4.5

IV Incapacitating systemic disease which is constantly life-threatening 25

V Moribund, not expected to survive 24 h with or without surgery 50

42. Theme: Sterilisation, disinfection and antiseptics A Autoclave B Ethylene oxide C Ionising radiation D Glutaraldehyde E Povidone-iodine. From the list above, choose the most appropriate agent to achieve sterilisation, disinfection or antisepsis for the following situations. Each answer may be used once, several times or not at all. Scenario 1 Surgical instruments (eg forceps, scissors) A - Autoclave« CORRECT ANSWER. An autoclave is a method utilising steam under pressure to sterilise surgical instruments and drapes. It destroys micro-organisms, viruses and spores. Scenario 2 Endoscopes (eg gastroscope) B - Ethylene oxide« YOUR ANSWER D - Glutaraldehyde« CORRECT ANSWER. Glutaraldehyde is a disinfecting agent. Its use does not ensure destruction of all micro-organisms and spores. It is used for disinfecting flexible endoscopes. Scenario 3 Suture materials C - Ionising radiation« YOUR ANSWER B - Ethylene oxide« CORRECT ANSWER. Ionising radiation is also used by manufacturers for sterilising disposable products and instruments but for smaller items such as sutures ethylene oxide is the most sensible option. Scenario 4 ‘Scrub up’ prior to surgery D - Glutaraldehyde« YOUR ANSWER

Page 39: Page | 1104

MyPasTest » MRCS A Online - Jan Exam 2015 14. Principles of Surgery in General; Perioperative Care (55Qs)

-------------------------------------------------------------------------------------------------------------------

---------------------------------------------------------------------------------------------------------------------------------------------------------------------------------------------------------------

Dr Mohammed Shamsul Islam Khan, Medical Officer, Clinical Neuro-Surgery, National Institute of Neuro-Sciences and Hospital

Sher-E-Bangla Nagar, Dhaka-1207, Bangladesh. Mobile: +880 1713 455 662, +880 1685 811979. E-mail: [email protected]

Page |

1142

E - Povidone-iodine« CORRECT ANSWER. Povidone-iodine is an antiseptic used for skin preparation during ‘scrubbing up' prior to surgery. Scenario 5 Surgical drapes E - Povidone-iodine« YOUR ANSWER A - Autoclave« CORRECT ANSWER. An autoclave is a method utilising steam under pressure to sterilise surgical instruments and drapes. It destroys micro-organisms, viruses and spores. Scenario 6 Urinary catheter F - « YOUR ANSWER C - Ionising radiation« CORRECT ANSWER. Ionising radiation is used by manufacturers for sterilising disposable products and instruments. 43. A 66-year-old woman presents at the pre-assessment clinic. She is due for elective hernia repair. Which of the following is an indication for a pre-operative chest X-Ray?

Age over 55« YOUR ANSWER

Allergic rhinitis

Known COPD« CORRECT ANSWER

Kyphosis

Previous reaction to a general anaesthetic. Pre-operative chest X-Ray is important in patients with a known cardiac or respiratory disorder, those with a malignancy or undergoing major thoracic or upper abdominal surgery. It is no longer necessary to obtain a CXR on age criteria although may be factored into the general picture of assessing the pre-operative patient. 44. Theme: Pre-operative management A Oral contraceptives containing oestrogen B Anabolic steroids C Antihypertensive drugs D Oral corticosteroids E Oral anticoagulants F Aspirin G IV heparin. For each of the statements below select the most likely answer from the list above. Each option may be used once, more than once or not at all. Scenario 1

Page 40: Page | 1104

MyPasTest » MRCS A Online - Jan Exam 2015 14. Principles of Surgery in General; Perioperative Care (55Qs)

-------------------------------------------------------------------------------------------------------------------

---------------------------------------------------------------------------------------------------------------------------------------------------------------------------------------------------------------

Dr Mohammed Shamsul Islam Khan, Medical Officer, Clinical Neuro-Surgery, National Institute of Neuro-Sciences and Hospital

Sher-E-Bangla Nagar, Dhaka-1207, Bangladesh. Mobile: +880 1713 455 662, +880 1685 811979. E-mail: [email protected]

Page |

1143

Inform the anaesthetist and continue therapy up to and after surgery if appropriate A - Oral contraceptives containing oestrogen« YOUR ANSWER C - Antihypertensive drugs« CORRECT ANSWER. Antihypertensive therapy is continued up to and after surgery unless the surgery is for treatment of the hypertension e.g. phaeochromocytoma or renal artery stenosis where it would only be continued if necessary. The anaesthetist must be informed to avoid drug interactions. Scenario 2 Discontinue 4 - 6 hours before surgery B - Anabolic steroids« YOUR ANSWER G - IV heparin« CORRECT ANSWER. IV heparin is discontinued 4 - 6 hours before surgery to allow blood clotting at and immediately after surgery. Scenario 3 Discontinue oral administration but replace with intravenous equivalent at induction C - Antihypertensive drugs« YOUR ANSWER D - Oral corticosteroids« CORRECT ANSWER. Long-term oral corticosteroid therapy requires intravenous steroid replacement to avoid sudden withdrawal of corticosteroid therapy resulting in adrenal insufficiency during and after surgery. Scenario 4 Discontinue and replace with intravenous or subcutaneous equivalent 48–72 hours before surgery D - Oral corticosteroids« YOUR ANSWER E - Oral anticoagulants« CORRECT ANSWER. Oral anticoagulants are discontinued and ‘replaced’ at least 48 hours preoperatively as with heparin. 45. THEME: Renal replacement therapy A Intermittent haemodialysis B Continuous venous haemodiafiltration C Continuous ambulatory peritoneal dialysis D Automated peritoneal dialysis. For each of the patients below, select the most appropriate type of renal replacement therapy from the list above. Each option may be used once, more than once or not at all. Scenario 1 A 50-year-old lady in acute renal failure secondary to sepsis from perforated appendicitis. She is 6 hours post-op appendectomy and has been anuric for more than 12 hours. She is normotensive with inotropic support. Her biochemical renal function is deteriorating and currently her urea is 12.5, her creatinine is 458 and her potassium is 3.6. A - Intermittent haemodialysis« YOUR ANSWER B - Continuous venous haemodiafiltration« CORRECT ANSWER.

Page 41: Page | 1104

MyPasTest » MRCS A Online - Jan Exam 2015 14. Principles of Surgery in General; Perioperative Care (55Qs)

-------------------------------------------------------------------------------------------------------------------

---------------------------------------------------------------------------------------------------------------------------------------------------------------------------------------------------------------

Dr Mohammed Shamsul Islam Khan, Medical Officer, Clinical Neuro-Surgery, National Institute of Neuro-Sciences and Hospital

Sher-E-Bangla Nagar, Dhaka-1207, Bangladesh. Mobile: +880 1713 455 662, +880 1685 811979. E-mail: [email protected]

Page |

1144

This is commonly used in unstable critically ill patients it is uncomplicated to commence in the intensive care unit, as it does not require fistula formation. 20–30% of patients with ARF who are haemodialysed become hypotensive, with huge associated osmotic shifts – disequilibrium syndrome. Many ICU patients are intolerant of such shifts. Moreover it appears that the haemodynamic changes that occur during haemodialysis (hypotension) may worsen the pre-existing renal injury by increasing the ischemic insult. Scenario 2 A 28-year-old obese lady with polycystic kidneys and end-stage chronic renal failure. She is awaiting a kidney transplant and only other significant medical history is that she had an emergency Caesarean section aged 24 years old resulting in a lower midline laparotomy scar. B - Continuous venous haemodiafiltration« YOUR ANSWER A - Intermittent haemodialysis« CORRECT ANSWER. This is the most efficient method of dialysis in the stable patient, large amounts of fluid can be removed and electrolyte abnormalities can be rapidly corrected. Obesity and risk of abdominal adhesions would make peritoneal dialysis a less preferred option. Scenario 3 A 58-year-old gentleman with diabetic nephropathy and retinopathy who has chronic renal failure requiring renal replacement therapy. He has ischaemic heart disease, needle phobia and has a job that involves travelling around the country. C - Continuous ambulatory peritoneal dialysis« CORRECT ANSWER. This is the most suitable of the options. It is ideal for patients with bleeding diatheses needle phobia and those with poor cardiac function who are unlikely to tolerate hypotension caused by haemodialysis. His busy job involving travel makes it unlikely that automated peritoneal dialysis would be an option. Scenario 4 A 67-year-old gentleman post-op has a difficult suprarenal aneurysm repair with a cross clamp time of approximately 4 hours. He has been anuric for 24 hours; his urea is 15 and his creatinine 250 and rising. He has been appropriately volume resuscitated. D - Automated peritoneal dialysis« YOUR ANSWER B - Continuous venous haemodiafiltration« CORRECT ANSWER. This gentleman is likely to have acute tubular necrosis secondary to his long supra-renal clamp time. He is post-op and is likely to benefit from continuous venous haemodiafiltration for the reasons given above. 46. An adolescent aged 14 is seen at her GP clinic requesting an abortion of an unwanted pregnancy. She does not wish her parents to know about the procedure. What is meant by Gillick competence?

A child cannot consent to their own medical treatment« YOUR ANSWER

A child of any age can consent to treatment if he or she fully understands the medical treatment proposed« CORRECT ANSWER

A child of any age can consent to treatment if he or she fully understands the medical treatment proposed but this must be supported by at least one parent / guardian

Page 42: Page | 1104

MyPasTest » MRCS A Online - Jan Exam 2015 14. Principles of Surgery in General; Perioperative Care (55Qs)

-------------------------------------------------------------------------------------------------------------------

---------------------------------------------------------------------------------------------------------------------------------------------------------------------------------------------------------------

Dr Mohammed Shamsul Islam Khan, Medical Officer, Clinical Neuro-Surgery, National Institute of Neuro-Sciences and Hospital

Sher-E-Bangla Nagar, Dhaka-1207, Bangladesh. Mobile: +880 1713 455 662, +880 1685 811979. E-mail: [email protected]

Page |

1145

A child of any age can consent to treatment if he or she fully understands the medical treatment proposed so long as it is supported by two doctors and is in the child's best interest

Parental rights outweigh the rights of a minor. Gillick competence states ‘as a matter of Law the parental right to determine whether or not their minor child below the age of 16 will have medical treatment terminates if and when the child achieves sufficient understanding and intelligence to understand fully what is proposed’. It is worth noting however that the Gillick test sets a very high threshold for capacity that would probably be unattainable by many adults. 47. A 46-year-old lady of afro-caribbean origin is seen in clinic. She is due to undergo a mastectomy for breast cancer. She is known to have thalassaemia trait. What blood picture would you expect her to have?

All of the following« YOUR ANSWER

Hypochromic microcytic« CORRECT ANSWER

Macrocytic aneamia

Normocytic normochromic

None of the above. Thalassaemia is an inherited blood disorder in which the body makes an abnormal form of haemoglobin. It leads to an excess destruction of red blood cells, as a result red blood cells will appear small and abnormally shaped. 48. A 39-year-old woman is seen in the pre-assessment clinic. Routine blood tests confirm that she has iron deficiency anaemia. Which of the following blood parameters reflect this?

Raised mean cell haemoglobin« YOUR ANSWER

Raised mean cell haemoglobin concentration

Raised serum ferritin

Reduced mean cell volume« CORRECT ANSWER

Macrocytic cells. Iron deficiency anaemia is the commonest cause of anaemia worldwide. Diagnosis is based on reduced levels of haemoglobin, mean cell volume, mean cell haemoglobin, mean cell haemoglobin concentration, serum ferritin and serum iron. Microcytic, hypochromic red cells can be found under the microscope. It would probably not require additional treatment and would not alter the surgical or anaesthetic plan. 49. Theme: Suture material A ‘0’ Vicryl B 3‘0’ PDS C 6‘0’ PDS D 6‘0’ Prolene E 2‘0’ Prolene F 2‘0’ Silk G 1 PDS H Stainless steel wire.

Page 43: Page | 1104

MyPasTest » MRCS A Online - Jan Exam 2015 14. Principles of Surgery in General; Perioperative Care (55Qs)

-------------------------------------------------------------------------------------------------------------------

---------------------------------------------------------------------------------------------------------------------------------------------------------------------------------------------------------------

Dr Mohammed Shamsul Islam Khan, Medical Officer, Clinical Neuro-Surgery, National Institute of Neuro-Sciences and Hospital

Sher-E-Bangla Nagar, Dhaka-1207, Bangladesh. Mobile: +880 1713 455 662, +880 1685 811979. E-mail: [email protected]

Page |

1146

For each of the following situations, select the most appropriate suture material for the procedure from the above list. Each option may be used once, more than once, or not at all. Scenario 1 Small bowel anastomosis. A - ‘0’ Vicryl « YOUR ANSWER B - 3‘0’ PDS « CORRECT ANSWER. Numerous suture materials can be used for bowel anastomoses. The most popular materials are the absorbable ones: PDS (polydioxanone) and Vicryl. Non-absorbable materials can also be used, including Ethibond. The size of the suture would vary from 2‘0’ to 4‘0’. Stapling techniques have become commonplace with the advantage of speed and a wider functional side-to-side anastomosis. There is no evidence that anastomotic leak rates are any higher between suturing and stapling. Scenario 2 Mid-line abdominal wound closure. B - 3‘0’ PDS « YOUR ANSWER G - 1 PDS« CORRECT ANSWER. The abdominal wall can be closed through a variety of techniques: layered closure or mass closure. The latter is quicker and currently the most popular technique in the UK. The suture material may be absorbable or non-absorbable, loop or non-loop. However, it must be strong and heavy (usually a 1 PDS or 1 Nylon). Scenario 3 Sternotomy closure. C - 6‘0’ PDS « YOUR ANSWER H - Stainless steel wire« CORRECT ANSWER. Sternotomy incisions in adults are closed with stainless steel wires. However, sternotomy incisions in paediatric patients are closed with a heavy PDS suture. Scenario 4 Distal end of a below-knee femoropopliteal bypass. D - 6‘0’ Prolene « CORRECT ANSWER. Arterial anastomoses are fashioned with prolene, an non-absorbable material, but gortex has also been used. For small calibre anastomoses fine material must be used, otherwise narrowing would occur. Scenario 5 Securing a prosthetic mesh for incisional hernia repair. E - 2‘0’ Prolene« CORRECT ANSWER. Prosthetic mesh techniques have become the accepted means for repairing incisional hernias. The aim is for the mesh to be secured in position and not migrate; therefore, a non-absorbable suture such as 2 ‘0’ prolene is most appropriate.

Page 44: Page | 1104

MyPasTest » MRCS A Online - Jan Exam 2015 14. Principles of Surgery in General; Perioperative Care (55Qs)

-------------------------------------------------------------------------------------------------------------------

---------------------------------------------------------------------------------------------------------------------------------------------------------------------------------------------------------------

Dr Mohammed Shamsul Islam Khan, Medical Officer, Clinical Neuro-Surgery, National Institute of Neuro-Sciences and Hospital

Sher-E-Bangla Nagar, Dhaka-1207, Bangladesh. Mobile: +880 1713 455 662, +880 1685 811979. E-mail: [email protected]

Page |

1147

50. A 46-year-old man is seen in clinic seeking an abdominoplasty following rapid weight loss. He currently weights 120 kg, down from 160 kg. Which of the following regarding obesity is correct?

Body mass index is a widely used means of estimating obesity in patients« YOUR ANSWER

BMI greater than 35 is consistent with 'morbid obesity'

BMI = Weight (kg)2 / height (m)

Obese patients do not normally require HDU / ITU care

Obesity itself does not contribute to complications. A normal BMI lies between 20-25, while a BMI of between 25-30 is classed as being 'overweight'; a BMI of between >30-40 is obese; and a BMI of over 40 is 'morbidly obese'. BMI is equal to Weight (kg) / height (m)2 . Obese patients encounter significantly greater complications, both intra and post-operatively. ITU care must be considered and beds booked prior to admission. 51. Theme: Acid–base balance pHPaCO2HCO3

- A 7.20 3.1 11 B 7.42 6.1 35 C 7.56 3.0 30. For each of the patients described below, select the most likely set of blood gas measurements from the list of options above. Each option may be used once, more than once, or not at all. Scenario 1 A 68-year-old woman with a history of abdominal pain with rebound tenderness and guarding for 72 hours. She exhibits signs of peritonitis and a WBC of 23 x 109/l. A - 7.20 3.1 11« CORRECT ANSWER A – pH 7.20, PaCO2 3.1, HCO3

- 11 Intestinal obstruction would lead to loss of fluid from the intravascular space and produce third-space loss. The presence of peritonitis and a grossly elevated WBC strongly suggest the presence of infarcted bowel. All these factors would produce a metabolic acidosis. Scenario 2 A 45-year-old woman who had a total abdominal hysterectomy for fibroids is now complaining of shortness of breath 7 days after surgery. There is no history of haemoptysis or pleuritic chest pain. B - 7.42 6.1 35« YOUR ANSWER C - 7.56 3.0 30« CORRECT ANSWER C – pH 7.56 PaCO2 3.0, HCO3

- 30. The history in the second case is suggestive of pulmonary emboli. Pulmonary emboli may occur at any stage after surgery, but most frequently after 5 days, and should be suspected in all cases of dyspnoea (earliest sign). The ensuing hyperventilation would reduce PCO2, shifting the Henderson–Hasselbach equation to the right (reducing H+), causing a respiratory alkalosis. Scenario 3

Page 45: Page | 1104

MyPasTest » MRCS A Online - Jan Exam 2015 14. Principles of Surgery in General; Perioperative Care (55Qs)

-------------------------------------------------------------------------------------------------------------------

---------------------------------------------------------------------------------------------------------------------------------------------------------------------------------------------------------------

Dr Mohammed Shamsul Islam Khan, Medical Officer, Clinical Neuro-Surgery, National Institute of Neuro-Sciences and Hospital

Sher-E-Bangla Nagar, Dhaka-1207, Bangladesh. Mobile: +880 1713 455 662, +880 1685 811979. E-mail: [email protected]

Page |

1148

A 75-year-old man had an uneventful abdominal aortic aneurysm (AAA) repair 10 days ago. He is known to smoke 30 cigarettes per day. C - 7.56 3.0 30« YOUR ANSWER B - 7.42 6.1 35« CORRECT ANSWER B – pH 7.42 PaCO2 6.1, HCO3

- 35 Patient 3 may not be taking deep enough breaths due to pain from his laparotomy, which would be exacerbated by COPD from his smoking. He will therefore have a respiratory acidosis due to retained carbon dioxide. Note there is a compensatory rise in the serum bicarbonate level. Normal values: pH 7.35–7.45 PaO2 >10.6 kPa PaCO2 4.7–6.0 kPa HCO3– 24–30 mmol/l 52. During a daycase operating list, a 56-year old gentleman, weighing 84 Kg, asks you to repair his direct left inguinal hernia under a local anaesthetic. You agree. What would be the most appropriate anaesthetic and dose to use with this gentleman?

100ml 0.25% Bupivacaine with 1: 200 000 Adrenaline« YOUR ANSWER

17ml of 1% Levobupivacaine

20 ml 1% Lignocaine and 20 ml 0.5% Bupivacaine with 1: 200 000 Adrenaline« CORRECT ANSWER

10 ml 2% Lignocaine with 1: 50 000 Adrenaline and 10 ml 0.25% Bupivacaine

30ml of 1% Lignocaine. Lignocaine has a fast onset of action with a maximum safe dose of 3mg/kg (or 7mg/kg with adrenaline). Bupivacaine and levobupivacaine have a slower onset but longer duration of action (8-10 hours) with a maximum safe dose of 2mg/kg (or 4mg/kg with adrenaline). The recommended local anaesthetic agent for an open inguinal hernia regional block is a 50:50 mixture of lignocaine and bupivacaine with the possibility of addition of adrenaline 1:200000. Further evidence for the efficacy of combined lignocaine, bupivacaine and adrenaline may be found at: http://www.ncbi.nlm.nih.gov/pubmed/18841412 For purposes of dose calculation, it is useful to remember that '1ml of 1% = 10mg'; therefore, in this question, 20ml of 1% lignocaine = 200mg lignocaine and 20ml of 0.5% bupivacaine = 100mg bupivacaine. 53. THEME: Clotting disorders A Factor VIII deficiency B Factors II, VII, IX, X deficiency C Antithrombin activation D Fibrinolysis E Antiplatelet antibodies F Prostaglandin inhibition.

Page 46: Page | 1104

MyPasTest » MRCS A Online - Jan Exam 2015 14. Principles of Surgery in General; Perioperative Care (55Qs)

-------------------------------------------------------------------------------------------------------------------

---------------------------------------------------------------------------------------------------------------------------------------------------------------------------------------------------------------

Dr Mohammed Shamsul Islam Khan, Medical Officer, Clinical Neuro-Surgery, National Institute of Neuro-Sciences and Hospital

Sher-E-Bangla Nagar, Dhaka-1207, Bangladesh. Mobile: +880 1713 455 662, +880 1685 811979. E-mail: [email protected]

Page |

1149

For each of the patients described below, select the single most likely diagnosis from the options listed above. Each option may be used once, more than once, or not at all. Scenario 1 A patient presents with bleeding after being given warfarin. A - Factor VIII deficiency« YOUR ANSWER B - Factors II, VII, IX, X deficiency« CORRECT ANSWER Warfarin impairs the production of Factors II, VII, IX and X. Scenario 2 A patient presents with bleeding after being given heparin. B - Factors II, VII, IX, X deficiency« YOUR ANSWER C - Antithrombin activation« CORRECT ANSWER. Heparin acts predominantly by facilitating antithrombin III and is reversed with protamine sulphate. Scenario 3 A man with an inherited condition presents with spontaneous bleeding. C - Antithrombin activation« YOUR ANSWER A - Factor VIII deficiency« CORRECT ANSWER. The sex-linked inherited condition causing spontaneous bleeding is most likely to be due to Factor VIII deficiency, haemophilia A. 54. Theme: Local anaesthetics: types & dosages A 9ml B 12ml C 18ml D 21ml E 27ml F 28ml G 56ml H 60ml I 84ml J 140 ml. The following scenarios all refer to issues concerning local anaesthesia. Please select the most appropriate answer from the above list. The items may be used once, more than once, or not at all. Scenario 1 You are asked to infiltrate a surgical wound with 0.25% bupivacaine. Select the maximum safe volume that can be used. The patient weighs 70 kg. A - 9ml« YOUR ANSWER G - 56ml« CORRECT ANSWER G – 56 ml

Page 47: Page | 1104

MyPasTest » MRCS A Online - Jan Exam 2015 14. Principles of Surgery in General; Perioperative Care (55Qs)

-------------------------------------------------------------------------------------------------------------------

---------------------------------------------------------------------------------------------------------------------------------------------------------------------------------------------------------------

Dr Mohammed Shamsul Islam Khan, Medical Officer, Clinical Neuro-Surgery, National Institute of Neuro-Sciences and Hospital

Sher-E-Bangla Nagar, Dhaka-1207, Bangladesh. Mobile: +880 1713 455 662, +880 1685 811979. E-mail: [email protected]

Page |

1150

A 0.25% solution contains 2.5 mg/ml bupivacaine (the number of mg/ml can easily be calculated by multiplying the percentage strength by 10). The maximum safe dose of bupivacaine is 2 mg/kg with or without adrenaline added. Therefore a 70-kg patient can have 140 mg of bupivacaine, which equates to 56 ml of a 0.25% solution (140 ÷ 2.5 = 56). Scenario 2 A 6-year-old child weighing 20 kg is scheduled for hernia repair. The anaesthetist wishes to supplement general anaesthesia with an ilioinguinal nerve block. He decides to use 0.5% lidocaine with adrenaline. Select the appropriate safe volume to be administered. B - 12ml« YOUR ANSWER F - 28ml« CORRECT ANSWER. F – 28 ml A 0.5% solution contains 5 mg/ml lidocaine. The maximum safe dose of lidocaine is 3 mg/kg without adrenaline added and 7 mg/kg with adrenaline. Therefore a 20-kg patient can have 140 mg of lidocaine with adrenaline, which equates to 28 ml of a 0.5% solution (140 ÷ 5 = 28). Scenario 3 A 72-year-old patient weighing 60 kg requires intravenous regional anaesthesia (a Bier’s block) of the upper limb before manipulation of a displaced Colles’ fracture. Select the maximum safe volume of the anaesthetic agent of choice (concentration 2%). C - 18ml« CORRECT ANSWER. Intravenous regional anaesthesia (IVRA) under tourniquet control is a simple method of providing short-term anaesthesia of the distal arm or leg, first described by August Bier in 1808. The drug of choice for IVRA is prilocaine because it is the least toxic local anaesthetic and has the largest therapeutic index. A 2% prilocaine solution contains 20 mg/ml prilocaine. The maximum safe dose of prilocaine is 6 mg/kg. Therefore a 60-kg patient can have 360 mg of prilocaine, which equates to 18 ml of a 2% solution. NB The pressure in the tourniquet must be constantly observed and maintained at = 50 mmHg above the patient’s systolic blood pressure. 55. Theme: Use of antibiotics A No antibiotics B Cefuroxime on induction C Cephalosporin on induction D Cefuroxime and metronidazole on induction and two further postoperative doses E Cefuroxime and metronidazole on induction and to continue until specified. From the list above, choose the most appropriate therapy for each of the following scenarios. Each answer may be used once, several times or not at all. Scenario 1 A 43-year-old female undergoing a laparoscopic cholecystectomy for acute cholecystitis. A - No antibiotics« YOUR ANSWER B - Cefuroxime on induction« CORRECT ANSWER.

Page 48: Page | 1104

MyPasTest » MRCS A Online - Jan Exam 2015 14. Principles of Surgery in General; Perioperative Care (55Qs)

-------------------------------------------------------------------------------------------------------------------

---------------------------------------------------------------------------------------------------------------------------------------------------------------------------------------------------------------

Dr Mohammed Shamsul Islam Khan, Medical Officer, Clinical Neuro-Surgery, National Institute of Neuro-Sciences and Hospital

Sher-E-Bangla Nagar, Dhaka-1207, Bangladesh. Mobile: +880 1713 455 662, +880 1685 811979. E-mail: [email protected]

Page |

1151

In procedures when the bowel is not opened, the risk of wound infection is 10%. Cefuroxime on induction is sufficient as the biliary tract rarely involves anaerobic contamination. Scenario 2 A 62-year-old male undergoing a procedure for varicose veins. B - Cefuroxime on induction« YOUR ANSWER A - No antibiotics« CORRECT ANSWER. This is a clean procedure without the opening of any mucosa. The risk of wound infection is 2%. No antibiotics are required. Scenario 3 A 78-year-old male who presents as an emergency with peritonitis. The working diagnosis is a perforated diverticulum. C - Cefuroxime and metronidazole on induction« YOUR ANSWER E - Cefuroxime and metronidazole on induction and to continue until specified« CORRECT ANSWER. Cefuroxime and metronidazole on induction and to continue until specified. This is a procedure where a contaminated field is encountered. Faecal peritonitis carries a wound infection rate of 40%. Antibiotics should be continued postoperatively until sepsis resolves. Scenario 4 A 17-year-old female undergoing an appendicectomy for acute appendicitis. D - Cefuroxime and metronidazole on induction and two further postoperative doses« CORRECT ANSWER. Cefuroxime and metronidazole on induction and two postoperative doses. An appendicectomy involves opening the bowel mucosa. If the appendix is inflamed then two further doses of antibiotics postoperatively are usually sufficient. If there is a gangrenous or perforated appendix then continuation of antibiotics is indicated. Scenario 5 A 48-year-old male undergoing repair of a right inguinal hernia. E - Cefuroxime and metronidazole on induction and to continue until specified« YOUR ANSWER A - No antibiotics« CORRECT ANSWER. An uncomplicated hernia repair should not involve breach of mucosa. Hernia repairs utilise a reinforcing mesh placement in the inguinal canal.

Editor:

Dr Mohammed Shamsul Islam Khan

MBBS (CMC); FCPS-II (Neuro-Surgery)

Medical Officer, Clinical Neuro-Surgery

National Institute of Neuro-Sciences and Hospital

Sher-E-Bangla Nagar, Dhaka-1207, Bangladesh.

Mobile: +880 1713 455 662, +880 1685 811979.

E-mail: [email protected]